Сварка нержавейки электродом в домашних условиях: чем и как лучше варить нержавейку в домашних условиях

Содержание

Сварка нержавейки для начинающих: электроды для сварки, технология работы инвертором и полуавтоматом

В данной статье рассмотрены основные вопросы, задаваемые начинающими сварщиками, по сварке коррозионостойких сталей и даны ответы на них.

Варите нержавейку легко и с удовольствием  ручной дуговой сваркой покрытым электродом,  неплавящимся вольфрамовым электродом в среде защитных газов и, конечно же полуавтоматическим инвертором плавящимся электродом!

Сварка ММА

Вопрос №1.

Варит ли инвертор нержавеющую сталь? Вчера решил испытать судьбу. Взял круглый бак от стиралки и отправился к знакомому у которого есть инвертор. Варить пытался электродом для нержавейки, диаметром 3 мм. Дуга скачет, невозможно работать. Если добавить ток, дуга обрывается. В баке прожоги металла.

Сварочный инвертор аврора

Ответ:

Для сварки коррозионостойких сталей нужен инвертор со встроенным осциллятором или с «хорошо выпрямленным» током. В паспорте, который прилагается к инвертору, обычно указывается на какие металлы он рассчитан. Но даже если вы не найдете в паспорте требуемую информацию, всегда можно выяснить все возможности аппарата в специализированном магазине.
Лучше всего подходит аргонодуговая сварка (в частности, популярностью  пользуется сварочник Aurora PRO INTER TIG 200 PULSE). АДС позволяет выполнять качественные швы на тонкостенных листах и деталях (трубах, например). Соединить же тонкий лист (до 3 …5 мм) ручной дуговой сваркой и получить качественный шов – это задача непростая.

Металлы, устойчивые к коррозии обычно содержат много хрома, а он, в свою очередь, образует с кислородом воздуха оксиды, что зачастую приводит к растрескиванию шва во времени. Образуется окалина. Поэтому важно, выполняя тонкие работы с изделиями, к которым предъявляются высокие требования, производить поддув аргоном обратной стороны шва.

Для работы штучным электродом с флюсующей обмазкой необходим опыт. Нужно поиграться с полярностью, а не просто работать по инструкции.

С толстостенными изделиями, как уже говорилось выше, обычно не возникает  проблем. Но так ли много изделий или конструкций из толстой нержавейки вы знаете? Правильно, все, что встречается – относительно тонкостенное, до 5 мм толщиной в сечении.

Когда варят  тонкостенную нерж, приходится уже выкручиваться:

  • Токи нужно устанавливать как можно меньше, дугу держать как можно короче.
  • Дуга зажигается в стороне, а затем постепенно подводится к свариваемой кромке.
  • Обращайте внимание на подключение клеммы заземления, в ее направлении будет действовать дутье дуги.

Что касается прожогов, для  толщин в 1 мм нужен электрод более тонкий, чем 3 мм. Для поджига трехмиллиметрового электрода нужен ток, который моментально прожжет тонкий лист. Электродом «тройка» сваривать и «черные» металлы непросто, а по нержавейке, да еще для того, чтобы учиться, нужно брать электрод 2 или 1,6 мм

Лучше всего сваривается нержавейка с пониженным содержанием углерода. В связи с тем, что стали с высоким содержанием хрома склонны к концентрации напряжений, которые на порядок превышают аналогичные напряжения в углеродистой стали, нужно избегать перепадов температур.

Рекомендуется предварительный подогрев  до 200 -350 градусов Цельсия.

Основные особенности,о которых нужно знать:

  • Коррозионостойкие металлы имеют крайне низкий показатель теплопроводности. Это означает , что тепло передается в окружающее воздушное пространство медленно, а скорость образования сварочной ванны увеличивается.  По этой причине сварку  выполняют на низких токах. Если для  углеродистой стали определенной толщины вам необходимо было установить на своем  аппарате 80А, то для аналогичной нержавейки вам потребуется 60А. Силу тока в среднем снижают на 25%.
  • линейные размеры  при нагреве испытывают существенные изменения, тому причиной немалое значение коэффициента термического расширения. Следствие — большие поводки деталей. Если речь идет о сварке толстостенных деталей в стык без зазора — гарантировано такое соединение потрещит.
    Причиной тому высокие напряжения такого соединения, которые возникают из-за неравномерного расширения металла. Сварку встык при больших толщинах производите с зазором.
  • Большое количество легирующих элементов увеличивает  электрическое сопротивление, поэтому при  ММА работают электродами не более 350 мм в длину.
  • Строго соблюдайте режимы термообработки, рекомендуемые для той или иной марки , из-за склонности к возникновению межкристаллитной коррозии.

Электроды ММА для нержавеющих сталей

Вопрос №2.


В гараже стоит инвертор для РДС (ММА). Есть работа по сварке нержавеющей стали. Подскажите, какие электроды подходят для такой работы, какие не подходят. Какие «подводные камни» сварки теми или иными электродами?

Ответ:

Выбор электродов для нержавейки, действительно, требует надлежащего подхода. Благо, ассортимент их довольно широкий. Наилучшими для коррозиестойких сталей на сегодняшний день являются электроды ОК61. 30. Выпускаются они шведской компанией ESAB и успешно используются при сварке 12Х18Н10, 12Х18Н10Т, 08Х18Н10 и т.д. ОК61.30 с рутиловым покрытием имеют легкий поджиг, держат уверено дугу, обеспечивают оптимальный прогрев, т.е. очень хорошо проявляют себя. Шлак отлично отделяется.

Отечественные электроды в основном идут с базисным покрытием. Они довольно капризны и требуют от сварщика определенного мастерства (имеют склонность к залипанию, при поджиге дуги нередко происходит отслоение покрытия, могут внезапно прекратить работать), но выполненный ими шов обеспечивает высокие коррозионные свойства. В продаже часто встречаются марки ЦЛ-11, либо ОЗЛ-8.

Вопрос №3
Как правильно варить ЦЛ-11?

Ответ:

 Как и ESAB ОК61.30 электроды ЦЛ-11 изготовлены для конструкций ответственного назначения из сталей, содержащих Cr и Ni, типа 12Х18Н10Т, 12Х18Н12Б и т.п., которые будут работать в непростых условиях, когда к ним предъявляют большие требования.

Швы, полученные ЦЛ-11 имеют высокую стойкость к образованию коррозии между кристаллами.

Електроды ЦЛ-11

Перед сварочными работами детали зачищают крацовочной щеткой до металлического блеска, удаляют грязь, масло, коррозию, которая несмотря на то, что нержавейка, может проявлять себя. Дугу нужно стараться поддерживать как можно короче, шов формировать неширокими валиками. Для электродов до 4 мм используют ток DC и обратную полярность. Варят в любых положениях кроме «от потолка к полу». Если диаметр четыре миллиметра и более – возникают сложности с прохождением швов на потолке и по вертикали.

Благодаря малому содержанию «вредных элементов» и небольшому газообразованию ЦЛ-11 дает шов устойчивый к обычной коррозии и между кристаллами.
В случае, если электроды долгое время провалялись в сыром помещении и набрали влаги, требуется термообработка около 200 градусов Цельсия в течении часа.

Мех.показатели:
Временное сопротивление разрыву, более 540Н/мм2
Относительное удлинение, более 20%
Ударная вязкость более 80 Дж/см2

Аналоги ОЗЛ-7;-8, ESAB OK61.

85, ОК61.30

Вопрос №4

Какой газ применяют для защиты шва?

Ответ:

Вольфрамовым электродом удобно варить тонкостенные листы. Швы качественные. Защита ванны — аргон 100%. Ничего другого для вольфрама придумывать не нужно. Единственный недостаток — низкий КПД по сравнению с полуавтоматической сваркой, потому что сварочную проволоку приходится держать левой рукой, подавая в сварочную ванну.

Вопрос №5

Сам сварке только учусь. Расскажите о сварке нержавейки полуавтоматом. Какой газ лучше применять для нее?

Ответ:

По всем теоретическим канонам сварку нержавейки производят в аргоне. Но на практике получается не совсем так, а точнее, немножко по — другому. При сварке в аргоне сварщики жалуются на большое разбрызгивание металла, нестабильную дугу. Не будем углубляться в возможные причины того, почему так происходит. Например, при сварке алюминия нужно использовать только аргон высокой чистоты (высокоочищенный), иначе возникают аналогичные проблемы, шов получается с раковинами, дефектами, в окалине, сварка затруднена. Таким образом для сварки нержавейки нужно использовать высокочистый аргон, но на практике готовят смесь аргона и углекислоты в соотношении 95-98% к 2-5%

. Во всяком случае все промышленные работы проводят в такой среде. Допускается заменить углекислоту на чистый кислород в некоторых случаях.
Варить в 100% углекислоте не рекомендуется, хотя жажда опытов толкает сварщиков на разнообразные эксперименты заканчиваются они снижением коррозионной стойкости шва. Углекислота лучше всего подходит для «черных» сталей (то бишь низко- и среднеуглеродистых), по какой причине, читайте в статье «Защита сварочной ванны»

Теперь о технологии. Практикуют 3 способа:

• Сварка короткой дугой – позволяет избежать проплавление металла при соединении тонких листов
• Струйный перенос – лучше всего использовать порошковую проволоку без газа

• Импульсный режим (присадочный материал подается порционно каплями малой величины) — наилучший способ, позволяет практически полностью избавиться от брызг и уменьшить расход проволоки.

Вопрос №6

Здравствуйте! Трудность в следующем: не выходит настроить скорость подачи проволоки полуавтомата. Свариваю нержавейку. Защитная среда углекислота. Шов получается низкокачественный, дугу рвет. При поджиге дуги проволока сгорает до горелки. Как настроить полуавтомат?

Ответ:

Трудность возникла из-за неправильно подобранных режимов сварки. При подборе режимов ориентируйтесь на 2 основных параметра: с какой скоростью подается проволока и каково напряжение на источнике питания.

Сварочный полуавтомат

Сначала выбирается с какой скоростью будет подаваться проволока. Выбирается скорость исходя из толщины изделия. Так же скорость связана с током. Чем скорость подачи выше, тем больше ток. Под скорость проволоки выставляют требуемое напряжение. Если напряжение низкое – поджиг дуги затруднен, при высоком напряжении проволока быстро сгорает до токопроводящей части и дуга обрывается.
Вам необходимо верно подобрать соотношение параметров скорости и напряжения. Только в таком случае вы получите шов, который будет соответствовать критериям качества.

Сварка нержавейки электродом – способы и особенности

Нержавеющая сталь является востребованным материалом благодаря своим свойствам (коррозионностойкости и долговечности), в связи с чем часто возникает необходимость ее сварить. Однако, есть тонкости при работе с нержавейкой, которые мы и рассмотрим в этой статье.

Сварка электродами по нержавейке

Варить коррозионностойкие стали непросто. Для получения качественного сварного шва достаточной прочности требуется несколько факторов:

  • достаточное количество опыта у сварщика;
  • правильный подбор сварочных материалов, в частности электрода.

Способы сварки нержавейки

Мы рассмотрим 2 способа сварки:

  1. Ручная электродом;
  2. Ручная аргоном.

Каждый из представленных ниже методов предполагает использование определенного оборудования и точно выбранных расходных материалов.

Ручная электродом

Качество сварного шва, полученного этим методом достаточное, чтобы этот метод сварки мог применяться как в быту, так и на производстве. Ручная сварка с применением электрода с покрытием считается универсальной и используется во всех отраслях.

Достоинства ММА-сварки:

  • простой и легкий процесс сварки;
  • высокая продолжительность работы аппаратов;
  • компактные агрегаты небольшим весом;
  • получение прочных сварных швов;
  • подходит для самостоятельного обучения этому методу.

От правильности выбора сварочных материалов зависит качество и надежность сварного шва.

При ручной сварке рекомендованы электроды следующих марок:

  1. ESAB OK 61.30 имеет высокую устойчивость к межкристаллитной коррозии и дает надежное сварное соединение. Шлаковый слой отпадает самостоятельно, что увеличивает скорость сварки.
  2. AG E 308L-16 подходит для металлов, эксплуатация которых происходит при низких и высоких температурах.
  3. ESAB OK 63.30 применяют для сварки металлов, контактирующих с агрессивной средой. Эти электроды можно применять при сварке на постоянном и переменном токе.

Для сварки данным методом надо устанавливать режим постоянного тока с обратной полярностью.

Ручная аргоном

Аргонодуговую сварку применяют для получения внешне красивых сварных швов. Этот способ хорошо себя зарекомендовал во время сварки очень тонких деталей.

Для сварки нержавеющей стали аргоном необходимо использовать вольфрамовые электроды. Если следовать этой технологии, то сварной шов непременно получится прочным и качественным, даже при выполнении сварочных работ в бытовых условиях. При сварке этим методом слоя шлака на швах и разбрызгивания металла не будет. Аргонодуговая сварка считается самым чистым способом соединения металлов.

Для данного метода подходит постоянный ток с прямой полярностью или переменный.

Таблица 1. Зависимость силы тока от толщины металла

Толщина металла, мм Вид и полярность Сила тока, А Диаметр электрода, мм
1 Постоянный 30-60 2
Переменный 35-75
1,5 Постоянный с прямой полярностью 40-75 2
Переменный 45-85
4 Постоянный с прямой полярностью 85-130 4

Особенности аргонодуговой сварки:

  • дуга разжигается бесконтактно, чтобы избежать попадания вольфрамового покрытия от электрода в уже расплавленный металл;
  • во время сварки нужно исключить колебания стержня. В противном случае нарушится защитный барьер в рабочей зоне и, как следствие, произойдет окисление шва.

Данный метод сварки позволяет снизить расход сварочных материалов. Необходимо после окончания сварочных работ продолжить подачу аргона в течение 10-15 секунд. Эти действия помогут защитить раскаленный электрод от активного окисления.

Сварка нержавейки электродом в домашних условиях

Для этого лучше всего выбрать сварочный инвертор. Для дома подойдет аппарат, работающий от сети 220В. Небольшие габаритные размеры устройства и малый вес позволяют более комфортно работать с ним и перемещать.

Основой популярности инверторов стали доступная цена и получаемое качество сварного шва. Это привело к тому, что сварочные аппараты инверторного типа стали лидерами по продажам.

Таблица 2. Параметры для настройки инвертора

Толщина металла, мм Сила тока, А Диаметр электрода, мм
1,5 40-60 2
3 75-85 3
4 90-100 3
6 140-150 4

Для сваривания применяют постоянный ток обратной полярности.

Последовательность действий при сварке инверторным аппаратом:

  1. Зачистить рабочую поверхность металла от ржавчины, масел, других загрязнений при помощи металлической щетки.
  2. Произвести разделку кромок напильником или болгаркой при необходимости (толщина металла должна быть больше 4 мм). Проводя эту процедуру, мы обеспечиваем высокий уровень проплавления и заполняемость сварочной ванны.
  3. Если свариваемый металл тонкий, то свариваемые края нужно плотно придвинуть друг к другу и прихватить их.
  4. Если свариваемый металл толще 7 мм, то мы прогреваем его до 150 С.
  5. Разжечь дугу.
  6. Провести сварку короткой дугой.
  7. В конце сварного шва требуется сделать “замок”, который предотвратит появление свищей и трещин.
  8. Дать изделию остыть.
  9. Затем убрать шлак со шва, после этого — зачистить.
  10. Отполировать и отшлифовать.

Сварка тонкой нержавейки электродом

Чтобы качественно сварить тонкий металл нужно иметь теоретическую базу знаний и достаточно опыта. Помимо этого нужно обратить внимание не только на правильный подбор электродов, но и верно выставленную силу тока.

Для сваривания тонкой нержавейки электродом требуется сила тока меньше на 20% по сравнению с обычной сталью.

Правильно подобранный сварочный электрод-половина успеха при сварке. Например, для толщины заготовки в 3 мм диаметр электрода должен составлять 3-4 мм.

Длина стержня не должна превышать 35 мм, а температура нагрева металла — 500 С.

Так же как и для сварки обычной нержавейки дома, для тонкой лучше применить инвертор.

Сварка тонкой нержавейки инвертором электродом имеет некоторые правила:

  • место сварки и сами заготовки не нагревать выше 150 С;
  • сварка должна проходить на небольших показателях тока на достаточно высокой скорости и желательно без колебания дуги во время сварки;
  • чтобы металл не смог перегреться и, как следствие, не прожегся, перед сваркой нужно подложить под заготовки металлические кусочки, которые отведут часть тепла;
  • для сварки металла, толщиной менее 3 мм, разделка кромок не требуется;
  • необходимо обеспечить зазор между заготовками, величиной 1-2 мм;
  • после сварки не надо резко охлаждать металл.

Если вы планируете использовать в работе электрод толщиной 3 мм, то необходимо выставить ток в 80 А.

Рассмотрим, какие электроды нужны для сварки тонкой нержавейки:

  • ЦЛ-11 – является одной из ходовых марок. Шов, полученный при помощи этого электрода, достаточно коррозионностойкий при неблагоприятных условиях.
  • ОК 63.20 используется для металла, имеющего контакт с жидкими агрессивными неокислительными средами при температуре до 350 С.

Сварка нержавейки в домашних условиях.

При принятии решения о сваривания нержавеющей стали, и изделий из нее в домашних условиях следует учитывать все особенности материала и подобрать оптимальный способ для выполнения этих работ. Нужно учитывать, что процесс по свариванию изделий из нержавеющих сталей требует очень большого труда.

Затрудняется процесс выполнения сварных работ образованием в процессе тугоплавких карбидов, а также повышению хрупкости металла, особенно на месте шва.

Также не стоит забывать о возможности развития межкристаллитной коррозии.В принципе по большому счету можно сваривать нержавейку любым из существующих способов сварки, будь то: аргоновая, аргоново-дуговая, точечная, холодная, импульсно дуговая и другие. Но на практике, конечно же, существует наиболее оптимальный и действенный способ не требующих через, чур, огромного затрачивания времени и сил при выполнении сварных работ. Лучше всего осуществлять, наплавку нержавейки применяя метод пайки металлов.

В любом случае какой-бы метод не был выбран необходимо обработать получившийся сварной шов на изделии или конструкции из нержавеющей стали. На поверхности сварного шва зачастую появляется пористый слой, образованный воздействием оксида хрома. Его легко укрепить при помощи травления и последующей обработки сварного шва.

При профессиональном выполнении травления шва практически в 100% удается убрать вредный оксидный слой и устранить зону с низким содержанием хрома. Для выполнения травления рекомендуется использовать смешение двух кислот: азотистой и фтористоводородной.

Для достижения наиболее хорошего результата перед выполнением сварных работ необходимо предварительно подготовиться к процессу сварных работ и учесть ряд требований:

— для избежание охрупчивания металла необходимо ограничить содержание ферритной кислоты в окружающем пространстве; — для не допускания коробления конструкций нужно увеличить длину прихватов и уменьшить расстояние между ними; — желательно сочетать в конструкции металлы с аустеничной структурой, это будет залогом выдерживания негативного влияния низких температур

 


как выполнить качественный шов в домашних условиях?

Сварка нержавейки, при которой пользуются аргоном как защитным газом, является одной из самых распространенных технологий получения качественных и надежных соединений деталей, изготовленных из такой стали.


Использование аргона при сварке нержавеющей стали позволяет получать сварные швы высокого качества

Прежде чем приступать к обучению этому процессу, следует познакомиться с характеристиками данного сплава, которые и делают его трудносвариваемым материалом.

Нержавеющая сталь является металлом, который успешно противостоит коррозионным процессам. Таким его делают легирующие добавки, основной из которых является хром (в отдельных марках нержавейки он может составлять до 20%). В различные виды такой стали могут также добавляться в качестве легирующих элементов титан, никель, молибден и др. Эти добавки, кроме антикоррозионных свойств, наделяют нержавейку и рядом других необходимых физико-механических характеристик.

Нержавеющая сталь, кроме исключительных антикоррозионных свойств, обладает поверхностью привлекательного внешнего вида. Именно поэтому ее часто даже не покрывают краской. Отсюда возникают дополнительные требования к качеству сварного шва: он должен быть не только надежным, но и аккуратным.

Выполнять сварочные работы с нержавейкой и получать соединения, удовлетворяющие самым строгим требованиям, может только специалист, обладающий не только необходимыми знаниями технологии, но и достаточным опытом работы в данной области. Это значит, что для обучения приемам сварки нержавеющей стали в среде аргона недостаточно просто посмотреть видео такого процесса – необходимо еще получить практические уроки.

Сварка электродом с использованием РДС-инвертора

Эта технология предусматривает использование электрода. Процесс характеризуется тем, что во время плавления стержня создается газошлаковая защита благодаря покрытию. Оно обладает видом шлаковой корки, разделяющей зону сварочной ванны и дуги от окружающего воздуха. Сварочное соединение возникает при расплавлении металла изделия и электродного стержня. Вы эту технологию можете знать по названию ММА, которая принята в международной практике.

Режим Pulse

Современное оборудование оснащено функцией выполнения работ в импульсном режиме. Его основное предназначение – соединение элементов различной толщины.

Как было сказано выше, данная технология помогает экономить расходный материал. Кроме того, сокращается время финишной очистки поверхности, благодаря низкому количеству брызг расплавленного металла.

Таким образом, можно сократить промежуточный этап механической шлифовки изделия, переходя к обработке кислотами и гелями, с целью удаления оксидного слоя и придания шву необходимой стойкости.

Где используется метод сварки с помощью инвертора

Если вы задались вопросом о том, как варить нержавейку инвертором, то должны знать, что технология может использоваться в любом случае, а пространственное положение на это никак влиять не будет. Вертикальные швы не обязательно получатся качественными, даже если у мастера есть определенный опыт.

Применение ручной дуговой сварки с помощью покрытых электродов имеет смысл при необходимости создания коротких швов, которые востребованы в мелкосерийном производстве. Этот метод актуален и при установке металлоконструкций при небольшом объеме работ.

Подготовительные работы

Сварка тонкой нержавейки аргоном требует специальной подготовки для получения качественного конечного результата. Есть несколько видов сварки нержавеющей стали. Сегодня наиболее популярными и востребованными считаются:

  • применение в работе покрытых электродов;
  • использование вольфрамового электрода;
  • сварка аргоном тонкого металла в режиме «полуавтомат» с использованием специальной нержавеющей проволоки.

Каждый из этих способов имеет свои особенности и нюансы. Чтобы выбрать оптимальный вариант для работы необходимо понимать, что вы хотите сделать и какой материал у вас для этого имеется. Настройка аргонной сварки для нержавеющей стали проводится в зависимости от материала и нюансов планируемой работы.

Особенности сварки нержавейки инвертором

Когда вам известен ответ на вопрос о том, можно ли варить нержавейку, вы можете выбрать метод. Если он заключается в использовании инвертора, то вы должны будете осуществить обработку и подготовку поверхностей к соединению. Она не отличается от той, которая используется в случае с низкоуглеродистой сталью.

С поверхности удаляются загрязнения. Предстоит поработать еще и над кромками. Для этого используются растворители. Эта операция позволяет удалить жир, воздействие которого ухудшает стабильность дуги. На обрабатываемую поверхность наносится препарат от налипания брызг.

Если перед вами, как и многими домашними мастерами, встал вопрос о том, как варить нержавейку инвертором в домашних условиях, то вы должны знать, что сварной стык должен обладать пазом. Благодаря этому удастся создать соответствующие условия для оптимальной усадки.

В процессе работы предстоит использовать ток обратной полярности. При выполнении сварки вы должны будете следить за тем, чтобы шов проплавился минимально. Электроды внушительного диаметра использовать не следует. Их применяют, когда есть необходимость сварить толстые поверхности.

При выборе электродов вы должны пользоваться специальной таблицей. Если допустить ошибку, то это станет причиной нарушения герметичности шва и повысит риск возникновения пор, раковин и микротрещин. В качестве причины здесь выступает закипание металла.

Варить сваркой нержавейку необходимо с использованием тока, значение которого на 20 % ниже по сравнению с тем, что используется для низколегированных сталей. Если вы хотите использовать инвертор для эксплуатации в быту или частном строительстве, то можно выставить диапазон вплоть до 160 А. Плавная регулировка позволит добиться максимальной точности тока сварки, что положительно скажется на качестве.

После завершения работы шов оставляется до момента остывания, что позволит высоколегированной стали противостоять коррозионным процессам. Проблема охлаждения решается с помощью медных прокладок. Когда в работе задействована аустенитная сталь, охладителем может выступить обычная вода.

Обучение

Перед началом работы необходимо подготовить все материалы и провести их обработку. Для начала рекомендуется обработать кромки деталей, которые вы планируете варить. Для обеспечения качественной усадки шва необходимо оставить небольшой зазор. Таким образом вы сможете сделать качественную работу, которая будет прилично выглядеть. Расход аргона при сварке нержавейки зависит от самого материала и количества работы.

Важно провести зачистку поверхности кромок. Для этого нужна стальная щетка. После этого поверхность кромки важно обработать растворителем. Для этого оптимально подойдет ацетон или авиационный бензин. Этот процесс проводится для удаления жира. Это обязательный этап. Если его пропустить, то устойчивость дуги будет ниже и в шве будут образовываться поры. Сварка пищевой нержавейки аргоном должна проводиться очень аккуратно.

Использование инвертора: подготовка инструментов и материалов

Теперь, когда вы больше не задаетесь вопросом о том, варят ли нержавейку, можете приступать к работам. При использовании инвертора необходимо подготовить инструменты и материалы, а именно:

  • сварочный инвертор;
  • растворитель;
  • средства индивидуальной защиты;
  • стальную щетку;
  • электроды.

Важно позаботиться о наличии зажимов-крокодилов для заземления. В арсенале должны присутствовать электрододержатели. Иногда эти элементы входят в комплект инвертора. Кабели должны иметь длину в 2 м или больше.

Как подобрать электроды

Если в работе вы планируете использовать метод ручной дуговой сварки, то должны позаботиться о правильном выборе электродов. Можно применить те, что имеют основное покрытие, среди них следует выделить:

  • СЭЗ ЗИО-8 d4,0.
  • ESAB FILARC 88S d3,2.
  • СЭЗ ЦТ-15 d5,0.

Они подойдут для сварки постоянным током. Роль покрытия играют карбонаты магния и кальция. Можно использовать электроды с рутиловым покрытием, среди них:

  • Lincoln Electric Omnia 46 D3,0.
  • ESAB OK 46.00 d3,0.
  • Межгосметиз Omnia 46 d3,0.

Обычно они изготавливаются на основе двуокиси титана и подходят для соединения с помощью переменного и постоянного тока обратной полярности.

Сварка аргоном

Перед вами может возникнуть вопрос о том, как варить нержавейку аргоном. Этот метод используется, когда металл очень тонкий или к соединению предъявляются особые требования по качеству.

Работать вольфрамовыми электродами в инертной среде лучше, если предстоит сваривать трубы, эксплуатирующиеся для перемещения газов и жидкостей под давлением. Сварка осуществляется в среде аргона переменным и постоянным током. В качестве присадочного вещества выступает проволока, которая обладает высоким уровнем легирования, чем отличается от основного металла.

Когда вам уже известно, чем варить нержавейку, следует взять электроды, чтобы выполнить ими работу без колебательных движений. Иначе вы можете нарушить защиту зоны сварки. Это станет причиной увеличения стоимости работ и повлечет окисление металла шва. Обратная сторона защищается поддувом аргона, но сталь не является критичной, чего нельзя сказать, например, о металле.

В сварочную ванну не должен попасть вольфрам. По этой причине следует использовать бесконтактный поджиг дуги. Ее зажигание может осуществляться на угольной или графитовой пластине, а после можно произвести перенос на основной металл.

Чем варить нержавейку, вы теперь знаете. Однако важно ознакомиться со всеми тонкостями работы. Прочитав инструкцию, сможете понять, что после завершения процедуры защитный газ не следует выключать сразу же. Это делается для уменьшения расхода вольфрамового электрода. Выключение следует осуществить через некоторое время, примерно через 15 секунд. Это предотвратит окисление электродов, которые будут оставаться горячими после работы. Кроме того, позволит значительно продлить срок службы.

Полезные советы

Чтобы досконально знать, как правильно сваривается аргоном нержавейка не стоит пренебрегать рекомендациями опытных сварщиков:

  1. Работа выполняется минимально короткой дугой, удерживая электрод на максимально близком расстоянии от металла, не затрагивая его. Длинная дуга не прогревает шов на всю глубину, поэтому его ширина увеличивается, а качество ухудшается.
  2. При проведении ручной сварки, чтобы не допустить окисления, проволоку подают плавно без рывков, не выводя ее из зоны действия аргона.
  3. О качестве проплава судят по форме наплывов образующихся, когда плавится присадочная проволока. Если они вытянуты вдоль шва — качество хорошее. Овальная или круглая форма свидетельствуют о недостаточном проплавлении.
  4. При подходе к концу шва величину тока нужно снижать. Резкий отрыв дуги с отведением горелки сопровождается снижением уровня защиты горячего шва, что сказывается на качестве соединения.

При правильном подходе аргонный метод не намного сложнее обычной сварки. Немного потренировавшись, любой желающий освоит его в кратчайшие сроки. Стоимость дополнительного оборудования и материалов окупится возможностью сваривать не только нержавейку, но также медь, бронзу, алюминий и их сплавы.

Свариваемость нержавеющей стали

Теперь вы знаете, как варить нержавейку электродом. Однако важно ознакомиться еще и со свариваемостью металла. Он является трудным вариантом, что зависит от многих параметров. На свариваемость влияют некоторые характеристики, например:

  • высокое значение линейного расширения;
  • сниженная теплопроводность;
  • высокое электрическое сопротивление;
  • склонность к потере антикоррозионных свойств.

Общая информация

Нержавеющий тип стали не подвержен коррозии. Этому металлу практически не страшна ржавчина даже без защитных спецвеществ. В этом основа популярности нержавеющей стали.

При правильном уходе она пригодна к использованию десятилетиями, а потому пользуется заслуженным спросом. Элементы хрома, никеля и титана, включенные в состав этой стали, улучшают ее физико-механические показатели.

Что тоже причина применения в разных областях.

К достоинствам нержавейки относится то, как она выглядит. Благодаря яркому, характерному блеску, вещи из такой стали зачастую не окрашивают.

А потому сварочные соединения на стальных изделиях необходимо делать не только надежными, но еще и сохраняющими внешний вид стали. Однако это лишь один из моментов работы со сталью.

Работать с нержавеющей сталью сложно. Те же характеристики состава, что защищают металл от ржавчины, делают его сварку непростой задачей. Эта статья о важных моментах сварочных работ с нержавейкой.

Ведь это задача, к которой требуется особая подготовка.

Факторы, влияющие на свариваемость

На свариваемость влияет еще нелитейная усадка. Это способствует тому, что в процессе сварки металл деформируется, как и после. Если между деталями внушительной толщины будет отсутствовать зазор, то могут возникнуть трещины. Перед тем как варить нержавейку полуавтоматом, вы должны узнать еще и о теплопроводности, которая значительно снижена с ней по сравнению с низкоуглеродистыми сталями.

Значение меньше в два раза, что вызывает концентрацию теплоты и способствует проявлению металлов в зоне сварки. Из-за этого возникает потребность в уменьшении силы тока на 20 % по сравнению с током для обычной стали.

Нельзя не упомянуть еще и о высоком электрическом сопротивлении, которое провоцирует нагрев электродов из высоколегированной стали. Для уменьшения отрицательного эффекта электроды изготавливаются с хромоникелевыми стержнями, которые обладают длиной не больше 350 мм.

Защита сварочного шва

Нержавеющая сталь отличается высокой чувствительностью к механической зачистке после завершения

процесса сварки. Зачистка подразумевает под собой снятие верхнего окисленного слоя, который как раз предназначен для защиты сварочного шва от ржавления. Восстанавливается окисленный слой только спустя 5-6 часов. Важно, чтобы в это время ничего не попадало в зону зачистки, что чаще всего просто невозможно. Но есть один способ, помогающий справиться с этой проблемой. После того, как механическая зачистка будет завершена, надо покрыть сталь специальным спреем, который состоит из пассивирующих присадок и синтетических масел.

Подводя итог, можно прийти к выводу, что прочность и качество швов при сварке нержавеющей стали зависит только от человека, выполняющего работу. Если подойти к выполнению всех требований со всей ответственностью и соблюдать все рекомендации, то результат оправдает ваши ожидания. Поэтому важно детально выполнить технологию сварки, подобрать хороший инвертор и купить качественные электроды.
[Всего голосов: 0 Средний: 0/5]

Как исключить межкристаллитную коррозию

Если работать предстоит с высокохромистой сталью, то вы должны знать, что она обладает одним важным свойством, выраженным в потере своих антикоррозионных характеристик. Это верно, если в процессе сварки использовать неправильный термический режим или аппарат для сварки. Это явление называется межкристаллитной коррозией, его природа заключается в том, что при температурах больше 500 °C по краям зерен формируется карбид хрома и железа. Они становятся очагами коррозионного растрескивания. С этим явлением борются разными методами. Например, с помощью быстрого охлаждения зоны работы.

Особенности сварки

Перед началом важно решить вопрос не только о том, чем варить нержавейку, но узнать еще и об особенностях проведения работ. Так, удельное электрическое сопротивление в 6 раз больше, а плавление на 100 °C меньше. Теплопроводность составляет 1/3 от этого показателя, свойственного углеродистому прокату.

Важно учитывать еще и тепловое расширение по длине, которое на 50 % больше.

Если вы не знаете, чем варить нержавейку, то должны учитывать, что использоваться могут самые разные методы. Так, если толщина материала больше 1,5 мм, то лучше воспользоваться ручной дуговой сваркой. А вот если речь идет о тонких листах и трубах, то лучше всего применить плавящиеся электроды в инертном газе.

Сварка нержавейки современным инвертором с электродами

Рассмотрим вопрос, какая же бывает электросварка, нержавеющей стали, покрытыми электродами.

Сварочный инвертор

Сварочный инвертор – это устройство являющееся источником питания для электрической дуги сварочного аппарата. Главная задача инвертора – обеспечить стабильную сварку, горение дуги и легкий поджиг. Сварочный процесс зависит от нескольких параметров.

Самый важным фактор – устойчивость к помехам и колебаниям. Сварочная дуга может питаться от выпрямителя, инвертора или трансформатора. Инверторный тип аппарата появился в 20 веке и стал невероятно популярным, благодаря своим рабочим показателям. Об известности инвертора можно судить по видео в сети Интернет. Каждый современный сварщик использует компактный инвертор.

Профессионалы, тщательно подбирают оборудование, для каждой работы

Принцип действия инвертора

Он достаточно прост: сетевое напряжение подается на выпрямитель, в котором преобразуется в постоянный ток с большей частотой. Сварочная дуга на постоянном токе становится более устойчивой и удобной для процесса сварки. Преимуществом использования инвертора для нержавейки, является уменьшение габаритов аппарата и улучшение динамических показателей дуги. Сварка инвертором повышает КПД работы, можно плавно регулировать параметры сварки и добиться минимального разбрызгивания металла. Если снять видео, то видно, что практически нет потерь металла во время работы.

Технология варки легированных сталей

Сварка нержавейки – это трудная задача в домашних условиях, зависящая от многих параметров. Согласно принятой классификации нержавеющая сталь относиться к типу высоколегированных сталей. В качестве легирующего элемента используется хром с содержанием около 20%. Кроме него в нержавейке могут присутствовать молибден, никель, марганец, титан и другие. Дополнительные металлы увеличивают антикоррозионные свойства нержавейки, а также придают ей другие физико-механические качества.

На сварку нержавейки оказывают влияние несколько свойств, которыми обладает высоколегированный металл:

  • Теплопроводность. У высоколегированных сталей теплопроводность ниже в 2 раза, чем у низкоуглеродистых металлов. Из-за этого сварка нержавейки происходит при пониженной силе тока на 20%.
  • Коэффициент линейного расширения больше, чем у других металлов. Поэтому необходим достаточный зазор между толстыми деталями, чтобы избежать деформаций. Если снять процесс сварки на видео, то видно как неравномерно расширяются детали из нержавейки.
  • Высокохромистые стали могут терять свои антикоррозионные свойства. Это явление назвали межкристаллической коррозией. Дело в том, что при температуре сварочных работ, по краям металлических зерен появляется карбид хрома и железа, с помощью которого в структуру металла проникает коррозия. С этим явлением борются различными способами. В основном быстро охлаждают место сварки, чтобы не допустить потери коррозионной стойкости.

При выборе сварочного аппарата, необходимо учитывать множество важных факторов

Перед началом сварки нержавейки инвертором в домашних условиях поверхность тонкой или толстой детали обрабатывается таким же методом, как и перед сваркой низкоуглеродистых сталей. За исключением одного момента – сварной стык должен быть с зазором, чтобы происходила свободная усадка. Поверхность деталей и кромки зачищают стальной щеткой и промывают растворителями (бензином или ацетоном). С помощью растворителя убирают жир, которые снижает устойчивость дуги.

Ручная сварка нержавейки в домашних условиях с помощью инвертора и покрытых электродов позволяет соединить детали с приемлемым качеством шва, а когда нет особых требований к соединению, то не надо заморачиваться с другими способами сварки. Чтобы определить какую марку электрода выбрать, достаточно обратиться к ГОСТу “Электроды покрытые для сварки высоколегированных сталей”. Если известна марка легированной стали, то легко по ГОСТу определить тип электрода. Например, это могут быть ЦП-11 или ОЗЛ-8.

Сварка нержавейки производится током обратной полярности. Во время работы нужно стараться меньше проплавлять шов и использовать электроды маленького диаметра. Но это справедливо для тонкой детали из легированной стали. Чем толще свариваемая поверхность, тем больше по диаметру выбирается электрод. Сила тока в инверторе должна быть понижена на 20% по сравнению со сваркой низколегированных сталей. Низкая теплопроводность электродов и высокое сопротивление при использовании тока высокого номинала может привести к отваливанию кусков покрытия электродов. По тем же самым причинам электроды плавятся быстрей во время сварки нержавейки. Это надо учитывать, если вы первый раз варите нержавеющую сталь электродом.

Чтобы сохранить коррозионную стойкость тонкой или толстой детали нержавейки, нужно производить охлаждение. Для этого используют медные подкладки, а если нержавейка является хромоникелевой аустенитной сталью, то можно использовать воду.

Как добиться качественного сварного шва

Сварка нержавейки требует особого мастерства. Даже полезное видео с техникой работы не сможет помочь, если вы не владеете определенными навыками. Разумеется, задача упрощается при сварке под аргоном, но и простым инвертором с прямым током можно добиться высокого качества шва. Самый главный показатель сварного шва нержавейки – это его герметичность. Из-за плохой теплопроводности металла, в сварном шве получаются раковины и поры. Они получаются в результате вскипания металла. Чтобы избежать этого явления, достаточно использовать специальные электроды для нержавейки.

Большинство изделий из нержавейки применяются в фармацевтической, пищевой отрасли или в домашних условиях. Это требует определенной обработки шва. Чтобы изделие выглядело привлекательно в жизни и на видео, сварной шов зачищают и полируют при помощи абразивов на основе оксида циркония или алюминия. После такой обработки нержавеющее изделие не требует дополнительного санитарного ухода.

Сварка нержавейки электродами и аргоном: особенности

Сварка нержавейки – трудоемкий, но вполне выполнимый процесс, совершаемый в домашних условиях. Сплавы с высоким процентным содержанием никеля (до 25%) с соответствующим количеством хрома с легкостью вступают в реакцию между собой. Состав таких сплавов прочно взаимодействует с остальными изделиями, исключая сплавы магния и алюминия.

Характеристика принципа работы инвертора

Выполняя сварочную работу, следует учесть, что при накаливании до 500-700С часть никелированных сплавов имеют свойство источать карбидное испарение. Уровень карбида напрямую зависит от рабочей продолжительности. При задействовании такого рода состава необходимо как можно меньше их использовать, дабы не превышать радиационный уровень в воздухе.

Инверторный процесс очень прост:

  1. Напряжение из сети поступает на выпрямительный модуль, который конвертируется в действие постоянного тока с высокой частотой.

  2. Дуга при сварке электродами с постоянной подачей тока приобретает наиболее устойчивый и удобный сварочный процесс.

  3. Плюсом в использовании инвертора для нержавейки является компактные габариты агрегата и высокие показатели динамики дуги.

  4. Сварка нержавейки инвертором с электродами увеличивает КПД при работе, также имеется возможность легко совершать регулировку параметров сварочного режима и иметь в итоге минимальное разбрызгивание сплава.

Способы сварки

Перед сваркой нержавеющей стали инвертором поверхностный тонкий или толстый участок деталей подвергают обработке таким же способом, что и до сварки стального листа с низкоуглеродистой структурой. В исключение входит один момент – это сварочный шов, который должен иметь зазор для свободной усадки.

Поверхностные участки элементов и кромка зачищаются щеткой из стали с дальнейшим обезжириванием растворителями (уайт-спирит или ацетон). Растворители уберут жировой налет, который снижает прочность дуги.

Сварка нержавейки ручным способом в домашних условиях с использованием инвертора обеспечивает соединение деталей с ровным качественным швом. И если особые требования к спайке не имеются, то не стоит применять другие виды сварок.

Для определения марки электродов достаточно просмотреть ГОСТ «Электроды с покрытием для сварки аргоном высоколегированного стального сплава». Если известна его марка, то по ГОСТ с легкостью можно определить тип электрода.

Сварка нержавейки осуществляется с подачей тока обратного полярного действия. В период сварки аргоном нужно меньше создавать проплавку шва нержавеющей стали и применять электроды минимальной толщины. Чем больше диаметр свариваемой части, тем больше диаметр электродного стержня. Напряжение в инверторе должно быть снижено до 20% в соотношении со спайкой низколегированного стального сплава.

Низкий показатель теплопроводности электродов и максимальное сопротивление при поступлении тока с высоким номиналом может спровоцировать раскол электродного покрытия.

Учитывая данный фактор, электроды поддаются расплавке в период сварочного процесса нержавейки быстрей. Об этом не стоит забывать, если сваривание нержавеющей стали электродами проводится впервые.

Для сохранения коррозионной устойчивости тонкого или толстого листа нержавеющей стали необходимо производить охлаждение. Для этого берут подкладку из меди, а если нержавейка имеет хромоникелевую аустенитную сталь, то используется вода.

Подготовка и создание сварочного шва

Кромки и саму поверхность нержавеющей стали перед сваркой аргоном чистят жесткой щеткой до блеска, после чего покрывают растворителем. Это нужно, чтобы повысить стойкость сварочной дуги и препятствовать появлению пор в швах нержавеющей стали.

Сварка нержавейки требует опытности и многолетнего стажа в области сварочных работ. Чтобы в итоге шов имел ровную поверхность и со временем не распался, добиться качественной работы можно аргоном с переменным напряжением. Из-за низкой теплопроводимости, после сварки аргоном появляются углубления и поры на местах стыка. Образуются они в режиме накаливания электродов. Чтобы этого не случилось, можно применять предназначенные для нержавеющей стали электроды.

Множество изделий из нержавейки применяются в фармацевтической, пищевой сфере или в быту. Процесс требует особенной технологии сварочного шва. Чтобы изделие имело достойный вид, шов при спайке нужно тщательно зачистить и отполировать с помощью абразивных реактивов в составе циркониевого или алюминиевого оксида. Обработав таким образом участок для сварочного процесса, нержавеющее изделие не нуждается в дополнительном санитарном уходе.

При работе с электродами важно не создавать колеблющих движений, которые приведут к неровностям и образованию пузырей. Необходимо, чтобы электроды не касались сварочного участка. Этого можно достичь с помощью бесконтактного метода розжига дуги. Помимо этого, розжиг дуги можно осуществлять на стержне графита или угля, после чего перенести ее на участок спайки. И, наконец, по завершению сваривания нержавейки не стоит прерывать подачу аргона в течение 15 секунд.

Похожие статьи

чем и как лучше варить нержавейку в домашних условиях

Популярные электроды для сварки нержавейки

К самым популярным электродам для нержавейки относят те, которые выпускают ведущие мировые производители. Использование брендовых изделий гарантирует получение качественного сварного шва.

ESAB


Эта шведская компания признанный лидер в разработке и изготовлении сварочного оборудования и расходных материалов, применяемого для работы с металлами разных типов.

ESAB OK 61.30

На ее предприятиях производят такие марки как:

  1. ОК 61.35 – их применяют для сварки особо ответственных конструкций, например, трубопроводов, работающих под давлением.
  2. ОК 67.72 — электроды, применяемые для сварки разнородных металлов.

ЦЛ 11


Электроды этой марки применяют для работы с такими сплавами как — 09Х18Н12Т, 12Х18Н10Т, Х14Г14Н3Т и их аналогами.

Ключевое достоинство этого расходного материала заключается в том, что шов, выполненный с этим электродом с успехом, противостоит межкристаллической коррозии.

МОНОЛИТ


Эта отечественная компания, которая выпускает электроды, применяемые для сварки углеродистых и нержавеющих сталей.

Электроды «Монолит»

Особенности сварки нержавеющей стали

Существует масса нюансов работы инвертором с нержавейкой. Требуется ответственно и внимательно подойти к каждому из них предварительно изучив все особенности.

Что из себя представляет инверторный аппарат

Каждый агрегат имеет свои конструктивные особенности, и перед началом любой работы требуется ознакомится с его инструкцией. Однако принцип действия и итоговый результат, при грамотном подходе, у всех схож.

Основным предназначением инвертора является преобразование переменного тока с напряжением 220В в постоянный, и увеличение его частоты одновременно со снижением высокого напряжения.

Когда сила тока будет переведена в необходимую и установленную на аппарате, можно производить сваривание заготовок.

Особенностью домашнего инвертора также является правило: им нельзя пользоваться очень долго. От перегрева кабеля и самих внутренних частей может случится авария. Именно поэтому этот прибор больше популярен для бытовых целей.

Настройка инвертора

Чтобы результат был такой, как нужно, необходимо правильно настроить значение агрегата до начала работы. Для сварки нержавейки подойдет абсолютно любой инвертор (даже самый менее мощный или самодельный).

Параметры будут зависеть непосредственно от толщины нержавеющей стали:

  • с толщиной металла 1.5мм требуется выбрать диаметр электрода 2мм, выставить минимальное напряжение 13V, выбрать силу тока 40А;
  • при 3мм потребуется электрод 3мм, напряжение 15V, а сила тока 75-85А;
  • если толщина металла 6мм, то электрод следует выбирать не менее 4мм, выставлять напряжение в 18V, а силу тока увеличить до 140-150А.

Обратите внимание! Все параметры указаны приблизительно. Перед началом сваривания нужной заготовки, неопытному сварщику необходимо потренироваться на подобном материале получив требующийся опыт.. https://www.youtube.com/embed/izmnVdZ0ZhM

Важные нюансы сваривания нержавейки

Несмотря на относительную несложность работы с нержавейкой, следует ознакомиться с необходимыми для успешной работы правилами и некоторыми нюансами:

  • лучше при покупке электродов выбирать не простые, а со специальной обмазкой, которая будет изолировать зону сварки и защищать сварочный материал от всевозможных внешних газов. Это необходимо так как нержавейка очень быстро реагирует с окружающей средой и если ее не защитить, то можно получить неровный шов, на который подействует углерод, образовавшийся от соединения кислорода и расплавленного железа;
  • нержавейка склонна подвергаться быстрому расплавлению если на нее будет долго воздействовать высокие температуры. Поэтому во время работы, желательно снижать мощность силы тока своего инвертора на 20% в отличие от сварки стали и вести работу в шахматном порядке. Это предотвратит деформацию металла;
  • одним из важнейших правил является грамотно выбранный электрод. Именно от его материала будет зависеть успешное завершение дела. Если присадочный материал выбран неверно это грозит образованию на стали микротрещин и нарушению герметичности.

Как выбрать электрод

Важно помнить, что для нержавеющей стали не подходят обычные электроды. Для такого металла они должен соответствовать определенным требованиям:

  • снижать затраченную на сварку мощность;
  • экономно расходоваться;
  • изготавливать шов ровно, без больших отложений шлака;
  • уменьшать длину и глубину зоны, подвергшейся термической обработке.

Для этих целей отлично подойдут электроды таких маркировок:

  • ОЗЛ-8, ЦП-11 – если требования к полученному шву минимальны;
  • ОК-45, МР-3 – более комфортен в процессе сварки. Шлак после них легко отчищается с поверхности;
  • ОК-61-30, ОК-67-45 – электроды, которые нужно использовать, когда необходимо сваривать между собой нержавейку с черным металлом;
  • неплавящиеся электроды – для сваривания нержавейки и алюминия;
  • самое высокое качество: ЭА-981-15, ОЗЛ-9-1. Они хорошо подойдут для сваривания жаропрочных сталей.

Чтобы технология была соблюдена полностью нужно помнить и об угле наклона. Он должен соответствовать приблизительно 75° к образовавшейся дуге.

Подготовка металла

Важно правильно подготовить материал, обработав его до процесса сварки. При работе с инвертором, нержавейку необходимо полностью зачистить, кромки разделать (если на то имеется необходимость)

Обратите внимание! Заготовки лучше всего зачищать специально предназначенной для этого щеткой.

Инверторный способ в домашних условиях, этапы работы

Инверторы для сварки нержавейки – это сварочные аппараты, которые отличаются компактными габаритами и небольшим весом. А если добавить их невысокую стоимость, получится отличный вариант сварки нержавейки в домашних условиях. Ток использовать постоянный с обратной полярностью.

Этапы инверторного способа рассмотрим поподробнее:

  • Прежде всего хорошенько очистить металлической щеткой поверхность от всех загрязнений.
  • Толщина краев заготовок не должна превышать 4-х мм, поэтому кромки необходимо разделать напильником или болгаркой.
  • Если металл тонкий, края плотно прижать друг к другу – сделать прихватки.
  • Если толщина металла выше 7 мм, заготовки нужно разогреть до 150°С. Для разогрева можно использовать паяльную лампу.
  • Поджечь дугу, произвести шов с соединением на короткой дуге.
  • Для профилактики образования трещин или свищей конец шва закончить «замком».
  • Остывание должно быть самостоятельным, никаких принудительных действий!
  • Очистить шов от шлаковой корки, отшлифовать и отполировать.


Свойства электродов для сварки нержавеющей стали.

Для инверторного способа используются электроды для сварки нержавейки с коррозионностойкими и жароустойчивыми свойствами:

  • ОЗЛ-6 – жаростойкие расходники, которые отличаются отличными техническими свойствами. При их использовании риск образования трещин и пор намного снижается.
  • АНО-27 – специальные расходники для работ при сильном морозе и при значительных нагрузках на конструкцию.

Выбор самого оптимального способа сварки нержавейки нужно делать с учетом трех моментов, все они про толщину стали:

  • Если толщина заготовок меньше 1,5 мм, варить лучше в аргоне с вольфрамовыми неплавящимися электродами. Способ – хоть ручной, хоть полуавтоматический.
  • При толщине в диапазоне от 1,5 мм до 3,0 мм самый лучший способ – электродуговой.
  • С толщиной больше 3-х мм – только электродуговой способ со струйным переносом металла.

Как варить нержавейку самостоятельно?

Любой из способов сварки деталей из высокоуглеродистых сталей подходит для соединения нержавейки в домашних условиях, но прочность в каждом случае будет разной. Наиболее популярным остается «союз» инвертора и электрода — ММА.

Оборудование, материалы, инструменты и защита

Подобная (созидательная) работа всегда подразумевает отдельный этап — подготовку всех элементов, аппаратуры и инструментов. Так как варить нержавейку достаточно сложно, то набор для этой работы потребуется большой. В него входит:

  1. Инверторный аппарат — прибор компактный и максимально эффективный. Кабель для подключения инвертора к электросети.
  2. Присадочные материалы. К ним относятся электроды, соответствующие классу нержавеющей стали. Если выбран вариант с аргоном, то потребуется баллон с газом, шланги для его подачи, газовая горелка.
  3. Инструменты для подготовки нержавейки. Это болгарка с шлифовальными кругами для этого вида металла, щетки (тоже именно для него), а также приспособления для надежной фиксации элементов будущей конструкции.
  4. Сварочные кабели, предназначенные для подачи тока в рабочую зону: это кабель «массы» (клеммы заземления) и кабель электродержателя. Главное требование к элементам — их достаточная длина, которая предотвратит перекручивания и другие подобные проблемы.

К обязательной экипировке сварщика относится:

  • защитный костюм, или плотная одежда, которая предотвратит контакт кожи с расплавленным металлом, защитит от высокой температуры;
  • перчатки, краги, маска, обязательно с темным стеклом;
  • ботинки из толстой кожи с такой же подошвой.

После завершения основной работы мастеру не помешают очки, которые предохранят глаза от «скачущего» шлака. Сварка — операция, которая делится на три отдельных процесса. Это подготовка, сама сварка и завершение работы.

Подготовка нержавеющей стали

Этот этап, состоящий из нескольких операций необходим любому материалу. Если говорить о «главной героине», то перед тем как варить нержавейку, мастеру нужно:

  • очистить соединяемые участки деталей — от загрязнений и оксидного слоя: для этого используют металлическую щетку, напильник, наждачную бумагу или инструмент со шлифовальными насадками;
  • подготовить к операции кромки: если толщина заготовок больше 4 мм, то их разделывают болгаркой, либо спиливают под углом;
  • предварительно нагреть элементы, чья толщина более 7 мм, температура зависит о марки материала;
  • уложить детали, соблюдая необходимый зазор между ними, его размер находят в справочнике;
  • сделать прихватки — короткие (точечные) швы, предотвращающие деформацию изделия.

Вместо механической очистки металла можно использовать химический метод. Как правило, в этом случае выбирают серную или соляную кислоту. Обработанные поверхности тщательно промывают. Перед началом сварки требуется обезжирить участки ацетоном либо авиационным бензином. После проведения всех подготовительных мероприятий можно начинать основную работу.

Сверка нержавеющей стали

Соединение деталей из нержавеющей стали выполняют так:

  1. Сначала подключают инвертор, создавая обратную полярность: кабель «массы» подключают на минус, кабель держателя на плюс. Такой метод позволяет снизить температуру плавления металла, а значит, дает возможность избежать прожога материала.
  2. После надежной фиксации заготовок выполняют предварительный этап — создают прихваточные швы. Их длина и шаг зависит от нескольких факторов — от толщины металла заготовок, от протяженности будущего соединения.

Если толщина зоны соединения большая (свыше 7 мм), то сначала ее разогревают до 150°, затем активизируют электрод, поджигая дугу. Сталь сваривают по короткой дуге. Чтобы предотвратить появление дефектов на трубах, делают «замок» — нахлест в 10-12 мм. После окончания сварки изделия оставляют остывать, минимальная пауза составляет 5 минут.

Завершающий этап

Он сводится к освобождению места соединения от образовавшегося шлака, окалины.Первую помеху удаляют небольшим (шлакоотбойным) молотком. После освобождения шва от несовершенств обязательно проверяют качество работы. Если «непровары» все-таки обнаруживают, то стыки вырезают, а сварку повторяют. Окончательную зачистку шва производят металлической щеткой, доводочным кругом, шлифовальным валиком и т. д.

Работа со сварочным оборудованием совсем не проста. Еще труднее операция с нержавеющей сталью, а рассказать о ней с помощью букв нереально. Ошибки неминуемы, поэтому только многочисленные тренировки на «подопытном» материале, а также советы мастеров помогут понять, а затем досконально изучить технологию.

Чтобы увидеть и узнать, как варить нержавейку правильно, лучше всего уделить немного времени популярному видео:

Полезная информация

Выбор изделий в соответствии с другими параметрами

Род тока, а также полярность его подключения являются важнейшими параметрами сварочных операций. Сварочные инверторы преимущественно вырабатывают постоянный ток, который может подключаться к заготовке и электроду по двум схемам.

  • Прямая полярность. При такой схеме плюс подключают к массе, а минус — к сварочному электроду.
  • Обратная полярность. Такая схема предполагает подключение минуса к массе, а плюса, соответственно, к держателю с электродом.

Если варить инвертором на прямой полярности, то соединяемые поверхности подвергаются значительному нагреву, чего не происходит при подключении полярности по обратной схеме. Именно поэтому выбор обратной полярности целесообразен в следующих ситуациях.

  • При выполнении сварки инвертором деталей небольшой толщины. Обратная полярность в таких случаях поможет защитить матриал от прожога.
  • На обратной полярности выполняется сварка деталей, выполненных из высоколегированных сталей, которые очень чувствительны к перегреву.

Работа инверторной сваркой

Прямую полярность, при использовании которой заготовка подвергается значительному нагреву, оптимально использовать для соединения материалов, отличающихся большой толщиной и массивностью.

При выполнении любых сварочных работ с использованием инвертора наиболее значимыми являются три параметра, которые взаимосвязаны друг с другом:

  • сила сварочного тока;
  • диаметр электрода;
  • толщина соединяемых деталей.

На выбор электродов толщина соединяемых деталей оказывает непосредственное влияние. При необходимости соединения тонких деталей (до 1,5 мм), ручная сварка не используется, для этой цели лучше подойдут полуавтоматические аппараты или же устройства, позволяющие выполнять сварку в защитной среде аргона.

Варианты положения электрода при сварке

Решая, какие электроды выбрать для сварки конструкций определенной толщины, можно руководствоваться следующими критериями:

  • для деталей, толщина которых составляет 2 мм, лучше всего подойдут электроды Ø 2,5 мм;
  • при соединении деталей толщиной 3 мм, следует выбирать электроды Ø 2,5–3 мм;
  • если толщина свариваемых деталей составляет 4–5 мм, то подойдут электроды Ø 3,2–4 мм;
  • детали толщиной 6–12 мм лучше всего варить электродами Ø 4–5 мм;
  • когда толщина превышает 13 мм, то оптимальным будет выбор электродов Ø 5 мм.

Сварочные электроды

Если же такой информации на упаковке не содержится, то можно руководствоваться следующими рекомендациями:

  • для сварки электродами Ø 2 мм следует устанавливать сварочный ток, сила которого составляет 55–65А;
  • для изделий Ø 2,5 мм используют ток 65–80А;
  • электроды Ø 3 мм — ток 70–130А;
  • для электродов Ø 4 мм выбирают сварочный ток 130–160 А;
  • изделия Ø 5 мм — ток 180–210 А;
  • 6-ми миллиметровыми электродами лучше варить на токе 210–240 А.

Как становится понятно из всего вышесказанного, для качественной сварки инвертором важен правильный выбор электродов по их диаметру. Также следует устанавливать оптимальную силу сварочного тока. Если, к примеру, вы соберетесь варить инвертором тонкий металл, используя электроды большого диаметра, или сила сварочного тока будет превышать допустимые значения, то в готовом шве могут образоваться поры, что значительно снизит его качественные характеристики.

Особенности сварки

Варить нержавеющую сталь возможно несколькими способами, но в каждом из них необходимо учитывать специфические особенности материала. Работа с легированной сталью отличается от низкоуглеродистой тем, что свариваемый металл позволяет формировать ровные швы, требующие минимальной обработки. Благодаря шлифовке и полировке можно получить идеальный вид поверхности, которую не нужно красить.

Но в том, как сваривать нержавейку, есть и свои трудности. Они заключаются в следующем:

  • Линейное расширение металла проявляется сильнее, чем у других видом стали. Из-за этого изделие значительно удлиняется во время нагрева от сварки, а по окончании процесса возвращается в первоначальную форму. Это создает два распространенных дефекта при сварке нержавейки. Во-первых, изделие сильно деформируется (образовываются волны, дугообразные прогибы), что портит внешний вид и требует правки геометрических форм. Во-вторых, происходит растяжение сварочного шва, который может не выдержать такого микродвижения и дать трещины. Правильная сварка нержавеющей стали подразумевает ведение дуги на меньших токах, чтобы минимизировать прогрев изделия, и подбор качественных присадочных материалов. В изделиях, которые толще 7 мм, применяется предварительный подогрев всей поверхности током.
  • В расплавленном состоянии нержавейка быстро взаимодействует с кислородом, находящимся в окружающем воздухе. Если вести сварку без защитного облака, то металл будет сильно пениться и шов не получится. Слабая защита сварочной ванны позволяет выполнить работу, но дает много пор. Поэтому заварить легированную сталь качественно можно только в хорошей защитной среде. Это обеспечивает специальная обмазка электродов или инертные газы.
  • Хорошая теплопроводность и низкая температура плавления материала создают еще одну трудность для сварки нержавеющей стали — выгорание легирующих элементов. Так, после нескольких месяцев, на свариваемом материале можно обнаружить следы коррозии. Чтобы предупредить этот дефект, необходимо выполнять шов немного быстрее, чем на низкоуглеродистой стали. Правильно установленная сила тока тоже играет важную роль.

Зная о вышеописанных свойствах металла можно выбирать верные режимы сварки и правильные расходные материалы, что позволит получить качественный результат.

Устройство электрода для сварки и наплавки

Основой любого электрода для сварки является металлический сердечник. При подаче на него электрического тока и образовании дуги происходит разогрев и плавление сердечника. На поверхности располагается обмазка, она при высокой температуре разогрева тоже расплавляется. Образует слой, предотвращающий попадание кислорода воздуха в зону плавления.

Для формирования обмазки используются несколько видов покрытия:

основной тип, используют для многократной проварки швов. Перед выполнением сварных работ требуется прокаливание или просушивание при температуре не менее 175…180 ⁰С;

кислотный тип применяют для сваривания заготовок, имеющих окисные пленки и ржавчину. Обязательно удалять окалину после завершения сварных работ. Прокаливание при температуре 180…220 ⁰С;

рутиловый тип, в составе присутствует окись титана. Для выполнения большинства работ с ручной электродуговой сваркой является самым подходящим вариантом. Требуется удаление остатков покрытия, имеет выраженную кислую реакцию. Краски на алкидной основе не могут удерживаться на поверхности шлака. При попадании во влажную среду нужно прокаливать при температуре не менее 185…200 ⁰С;

целлюлозный тип, в составе присутствуют органические материалы (кукурузная или древесная мука, целлюлоза, смолы органического происхождения). Маркируют такие электроды Э42…Э50 с разными буквенными обозначениями. Эксплуатируются на постоянном токе. Прокаливание при температуре не более 110…120 ⁰С.

Для инверторных аппаратов рекомендуют использовать кислотные и рутиловые электроды. Основной тип применяют редко, Трудно удерживается дуга. Капризное поведение не позволяет получать качественный сварной шов.

В быту использование расходного материала с целлюлозным покрытием также ограничено, цена довольно высокая.

Толщина покрытия

На практике толщина покрытия на поверхности электродов определяется в мм:

  1. 0,8…1,8 – тонкое покрытие. Используются максимально широко. До 90 % выпускаемой продукции имеет именно такую толщину.
  2. 2,0…3,6 – средняя толщина. Такие электроды используют для сваривания ответственных деталей, рассчитанных на динамическое нагружение с резкопеременной нагрузкой.
  3. 4,0…6,0 – большая толщина. Электроды используют для ответственных работ – сварки трубопроводов с высоким давлением внутри. При транспортировании агрессивной среды.

Материал сердечника

Для изготовления большинства электродов используется низкоуглеродистая сталь. В ней присутствует не более 0,72…0,78 % углерода. Металл:

  1. Сравнительно легко расплавляется. Температура в зоне дуги находится на уровне 1750…1950 ⁰С.
  2. Расплав заполняет предоставляемое ему пространство. Он затекает в полости, образующиеся при нагревании.
  3. Нагрев близок к температуре кипения, поэтому расплавляемая обмазка понижает температуру расплава.

Ковкий чугун используют для сварки высокоуглеродистых сталей и чугунов. Однако, есть особенности при сварке серого чугуна. Нужно предварительно нагревать материалы до аустенитного состояния (730…850 ⁰С). После завершения процесса нужно обеспечить медленное охлаждение. Если произойдет быстрое охлаждение, тогда вблизи зон сварного шва формируется белый чугун. Он обладает высокой хладноломкостью.

Марганцевая сталь применяется для сварки легированных материалов. Используют основные и целлюлозные покрытия.

Нержавеющая сталь используется для сварки трубопроводов из нержавейки. Применяют и сплавы, содержащие марганец и медь.

Специальные типы электродов

Для сваривания нержавейки и высоколегированных сталей используют электроды, которые не сгорают в процессе сварки. Они только создают дугу. Здесь используются присадочные проволоки, которые расплавляются и попадают в зону нагрева.

В таких технологиях используют титан. Он выдерживает нагрев выше 2200 ⁰С. Присадочные проволоки изготавливают из нержавеющей стали 18ХН9Т. Температура плавления составляет 1340…1380 ⁰С.

При сваривании легированных изделий в качестве присадочного материала применяют проволоки ХВС, ХВГ, ХС12, ХН9Т и другие. Они имеют температуру плавления на уровне 1420…1510 ⁰С.

Для тугоплавких сталей применяют присадочные прутки из ХНГ, 12ХВ10Т, Т10ХВ. Эти сплавы плавятся при температуре 1670…1820 ⁰С.

Особенности сварки нержавейки

Нержавеющая сталь сложная для сваривания вследствие повышенного содержания хрома — в сплаве его от 13 до 30%. При соединении с кислородом, хром образует оксидную пленку, которая препятствует сплавлению металла в сварочной ванне. Низкая теплопроводность металла вызывает перегрев в зоне шва и частичное изменение структуры материала, что приводит к снижению прочности.

Но бороться с этими трудностями вполне возможно, просто необходимо помнить об особенностях металла и выбирать оптимальный режим работы.

Важен и второй вопрос — чем лучше всего варить нержавейку в домашних условиях? Однозначного ответа здесь нет. Все зависит от марки стали и опыта сварщика. Если есть выбор, то лучше всего выбрать инверторный аппарат, как самый удобный в использовании и обладающий широким диапазоном точных настроек.

В бытовых условиях чаще всего используется сварка покрытыми электродами, но подходят не все виды. Необходимо выбирать только электроды с основным или рутиловым покрытием. Если ориентироваться на марки, то покупать следует ОЗЛ-8, НИАТ-1, ЦЛ-11 или их зарубежные аналоги. В каждом магазине, торгующим сварочным оборудованием, вам подскажут, какие электроды для какой стали подходят лучше всего.

Наиболее распространенными марками стали, с которыми приходится встречаться домашнему мастеру, являются AISI 304, 304L, 316L и 321. Аналогами по ГОСТ выступают 08Х18Н10, 03Х18Н11, 03Х17Н14М3 и 12Х18Н10Т. Именно такие металлы используются для посуды, изготовления труб и листов, из которых делают ворота, ограды и другие декоративные архитектурные элементы.

Настраивается аппарат на обратную полярность (+ на электроде) и сила тока выставляется на процентов 20-25 ниже, чем для сварки обычной стали такой же толщины. Также следует учесть тот факт, что электрическое сопротивление нержавеющей стали ощутимо выше, чем обычной. Электроды с низколегированным стержнем могут перегреваться и разрушаться в процессе работы.

https://youtube.com/watch?v=Zngv3j_zh5g

Под свариваемые листы нержавейки необходимо установить медную подложку, чтобы она отводила тепло из зоны сваривания и не возникало перегрева и металл не изменял своей структуры. Также не следует стыковать кромки вплотную — тепловой коэффициент расширения нержавеющей стали достаточно высокий, поэтому при охлаждении шов может покрываться микротрещинами. Зазор не должен превышать 2 мм. Электрод ведут вдоль прямой линии, колебательная траектория при сварке нержавейки не применяется.

Перед тем, как варить нержавейку покрытым электродом в домашних условиях, позаботьтесь о наличии всего необходимого для подготовки металла к работе и финишной обработки шва. Подготовка заключается в тщательной очистке зоны шва от пыли, грязи и следов технических жидкостей. Если есть возможность — пройдитесь зачистным кругом болгарки или мелкой наждачной бумагой. Затем необходимо промыть поверхность ацетоном или чистым бензином для удаления остатков масел и жиров.

Где чаще всего применяется метод

Области, в которых применяется сварка нержавейки инвертором, обширны за счет мобильности инвертора. Он не привязан к единому месту, поэтому работы могут выполняться как в домашних условиях, так и на производстве.

Сварка нержавеющей стали электродами будет полезна только при создании коротких швов. РДС востребована в следующих видах работ:

  •  Изготовление деталей в малых масштабах
  •  Установка металлоконструкций. Имейте ввиду, что данный вид сварки возможен при условии, что объем запланированных работ небольшой
  •  Сооружение прихваток во время установки конструкций под сварочные работы
  • Устранение дефектов на небольших участках шва
  • Наплавка

Резюмируя вышесказанное, стоит еще раз отметить, что сварка нержавейки электродом используется в случае, если предстоящие работы не имеет большого масштаба. Поэтому инверторное соединение нержавеющих сталей подходит для использования в личных целях, и в целях производства на малых участках. Соединяемыми элементами выступают металлические конструкции, предметы из нержавеющей стали или трубы.

Если вы все же намерены выполнить сварку нержавейки с помощью электродов, то последовательность выполнения работ описана ниже.

Настройка сварочного аппарата

Для сварки нержавеющей стали электродами придерживаются определенного режима работы. Чтобы сварить 4 мм заготовки, нужен аппарат, выдающий 100 А с напряжением 16 В. Диапазон сварки более тонких деталей:

Толщина заготовки, ммДиапазон силы тока, АРекомендуемое напряжение, В
130 – 4012
1,540 – 6013
2 – 3в пределах 8014 – 15

Диаметр электрода должен быть меньше толщины заготовки, сталь до 3 мм варят двойкой, 4 мм – 3-х мм стержнями.

При соблюдении всех технологических тонкостей сварки легированных металлов можно получить достаточно прочное соединение в домашних условиях. Для реставрации труб, емкостей, рассчитанных на высокое давление, лучше прибегнуть к услугам профессионалов.

Переменным или постоянным током

Сваривание переменным и постоянным током обладает своими особенными характеристиками.

Основные преимущества постоянного напряжения: экономия сварочных материалов за счет низкого уровня разбрызгивания; комфорт и легкость проводимых работ; качественный шов; высокая производительность сварки; отсутствие непроверенных участков. Недостатком является высокая стоимость оборудования, способного выдавать постоянный ток. Подробнее здесь.

Главные достоинства переменного тока: легкость и доступная цена оснащения, работающего на переменке; удобство проведения сварочных работ; гарантия качественного соединения. Основные минусы: меньшая стабильность дуги; большое количество брызг способствует значительному расходу материалов. Подробности тут.

Коррозионностойкие стали можно сваривать различными способами. Однако, чаще всего, для сварки нержавейки используются два метода соединения:

  1. Ручное сваривание покрытыми электродами.
  2. Сварка вольфрамовым электродом в среде защитных газов.

В зависимости от метода сварки используется различный вид напряжения, а соответственно применяются электроды, подходящие для переменного или постоянного тока.

Электроды постоянного тока по нержавейке

Приступая к работе мастер должен решить какими электродами можно варить нержавейку. Сварочные материалы с обмазкой без особых проблем обеспечивают оптимальное качество соединения. Ручное сваривание осуществляется, как правило, постоянным напряжением обратной полярности. Поэтому используются нержавеющие электроды следующих марок:ЦЛ-11 является одной из самых популярных марок среди сварщиков; используется для работы со сталями с высоким содержанием хрома и никеля. Шов, наплавленный с помощью данных расходников, обладает несколькими преимуществами: прочность; пластичность; аккуратность; достаточно высокий уровень ударной вязкости; отсутствие разбрызгивания.

Электроды ОЗЛ-8 предназначены для сварки конструкций, которые будут эксплуатироваться в условиях высоких температур – до 1000°С. При это достоинства данной марки во многом схожи с ЦЛ-11.

НЖ-13 успешно используются для сваривания деталей из пищевой стали. Расходники данной марки отлично сваривают сплавы, где присутствуют хром, никель и молибден. Главная отличительная особенность таких электродов – образование тонкого слоя шлаковой корки, которая отделяется самопроизвольно.

Электроды НИИ-48Г.

Ниже приведен перечень ещё нескольких востребованных электродов по нержавеющим сталям:

ЗИО-8 предназначены для жаростойких коррозионностойких сталей.

Электроды НИИ-48Г используются для работы с ответственными конструкциями.

ОЗЛ-17У подойдут для нержавейки, работающей в средах, где присутствуют серная или фосфорная кислоты.

В соответствующем разделе представлены остальные марки электродов для сварки нержавейки.

Электроды для переменного тока для нержавейки

Не все исполнители располагают оснащением, работающим на постоянном напряжении. Из-за чего возникает вопрос: можно ли варить переменным током нержавейку?

Есть такие электроды, например, это марки ОЗЛ-14, ЛЭЗ-8, ЦТ-50, ЭА-400, ОЗЛ-14А, Н-48, АНВ-36 и другие.
Сваривание вольфрамовыми электродами (на картинке) в среде газов также можно проводить переменным током прямой полярности. Данный метод соединения применяется в следующих случаях:

  • сваривание тонкостенных изделий;
  • повышенные требования к сварочному шву.

Данные сведения помогут исполнителю любого уровня определить какие электроды для сварки нержавейки переменным током следует использовать при решении конкретных задач.

В качестве вывода, следует отметить, что электроды для нержавейки переменного тока менее востребованы. Данный факт обусловлен меньшей популярностью переменного напряжения по сравнению с постоянным.

Постоянка обладает большим спектром достоинств и используется профессионалами намного чаще.

Дизайн кабинета: основные правила современного оформления

Соединение нержавейки и черного металла вполне возможно. Но, этот процесс сопряжён с определенными сложностями. Все дело в том, что у этих металлов разная структура. Для выполнения этой операции можно использовать три метода:

  • сваривание с применением расходных материалов с покрытием;
  • сваривание неплавящимися стержнями из вольфрама;
  • сваривание под защитным газом, как правило, для этого применяют аргон или газовые смеси на его основе.

Для сваривания разнородных металлов используют марку ОЗЛ-312. Для выполнения сборки ответственных конструкций применяют ЭА-395/9. Стержни для сварки нержавеющей стали марки ОЗЛ-312 подходят для сварки сталей с неопознанным составом.

Но, как показывает практика, оптимального качества шва лучше, чем соединение заготовок под защитой газов не придумали. Газ, в этом процессе исполняет роль защиты сварной ванны от воздействия атмосферы, в частности от азота и кислорода. При выполнении сварки аргоном, существует одна тонкость. Для обеспечения качества сварки применяют сварочный пруток, который необходимо держать строго под углом 90 ⁰ к обрабатываемым поверхностям.

На основании вышеизложенного можно сделать следующее заключение – для выполнения сварки разнородных металлов используют материалы широкого применения.

3 Стандартные методы сварки нержавеющей стали

Процесс сварки нержавеющей стали варьируется в зависимости от толщины и отделки материала, а также использования готового продукта. Хотя существует множество методов сварки нержавеющей стали, есть три, которые чаще всего используются сварщиками в Соединенных Штатах. К этим методам сварки нержавеющей стали относятся сварка TIG, контактная сварка и сварка MIG.
Это сварка TIG, контактная сварка и сварка MIG.Продолжайте читать, чтобы узнать больше о каждом.

1. Сварка TIG или газо-вольфрамовая дуговая сварка

Предлагая высокое качество, универсальность и долговечность, TIG является наиболее часто используемым процессом сварки нержавеющей стали. Этот процесс сварки обеспечивает низкое тепловложение, что делает его идеальным для обработки тонких материалов. Газ аргон часто смешивают с другими газами, в зависимости от потребностей конкретного проекта, включая гелий, водород и азот. Чтобы предотвратить окисление и повысить устойчивость к коррозии, можно использовать процесс односторонней сварки, создавая инертную газовую защиту между внутренними и внешними сварными швами.

2. Контактная или точечная сварка

Контактная или «точечная» сварка, как ее часто называют, — один из самых экономичных видов сварки. Оборудование для контактной сварки (RW) невероятно универсально, что означает, что его можно использовать как в небольших, так и в крупных проектах.

RW использует электрический ток для нагрева истертых металлических кромок и их склейки. Этот тип сварки исключительно эффективен для металла с низкой температурой плавления, поскольку его можно адаптировать для предотвращения деформации металла.

3. Сварка МИГ или газовая сварка металлов переменным током

Сварка

MIG — это полуавтоматический процесс, который при правильном выполнении обеспечивает прочное соединение двух кусков нержавеющей стали. В этом процессе используется защитный газ, богатый аргоном, и сплошной проволочный электрод.

Сварка

MIG популярна, потому что позволяет сварщику использовать импульсный источник тока, который может облегчить сварку труднодоступных мест на сложных проектах из нержавеющей стали. Смеси других газов, в том числе с гелием, кислородом и углекислым газом, часто используются для стабилизации дуги и улучшения качества сварного шва.

Какой метод сварки нержавеющей стали лучше всего?

Выбор правильного метода сварки нержавеющей стали на самом деле зависит от того, какие качества вы ищете. Если вы ищете более доступный сварной шов, то лучше всего подойдет точечная сварка. Но если материал, с которым мы работаем, тонкий, то лучшим выбором может быть сварка TIG или газо-вольфрамовая дуга.

В All-Type Welding and Fabrication, Inc. наша команда экспертов по сварке оценит материалы, возможности и стиль отделки, которые вы хотите использовать для каждого проекта, чтобы определить, какой метод сварки будет наиболее эффективным для данной задачи.

Обладая обширными знаниями в области сварки и многолетним опытом работы, компания ATWF может выбрать и реализовать для вас лучший метод сварки нержавеющей стали. Свяжитесь с нами сегодня, чтобы получить расценки, узнать больше о нашем процессе и получить все ответы, которые вы ищете.

Как сваривать нержавеющую сталь

Если вы хотите сваривать нержавеющую сталь , вам следует знать несколько вещей. Во-первых, нержавеющая сталь — это не один металл, а семейство металлов.Внешний слой из оксида хрома делает их устойчивыми к ржавчине. Также они бывают разной толщины. Перед тем, как начать, вы должны точно знать, с каким типом нержавеющей стали вы имеете дело. Во-вторых, нержавеющую сталь труднее сваривать, чем другие металлы, потому что она коробится и деформируется при высоких температурах. Это может повлиять на его прочность и внешний вид. Электродуговая сварка используется, но существует несколько разных видов, поэтому вы должны знать, какой из них подходит для вашего проекта. Наиболее распространенными типами сварки нержавеющей стали являются Stick (SMAW), Tig (GTAW) и Mig FCAW или GMAW.)

Шаг 1 — Безопасность

Как и при любой сварке, безопасность очень важна. При сварке образуются пары, искры и дым, которые могут быть опасными. Не экономьте на средствах безопасности, вы можете пожалеть об этом. Убедитесь, что поблизости нет ничего легковоспламеняющегося. Ищите лужи масла, обрывки, тряпки и все, что может воспламениться. Убедитесь, что есть надлежащая вентиляция. Не начинайте, пока полностью не разберетесь в своем оборудовании.

Шаг 2 — Планируйте вперед

Перед началом работы убедитесь, что детали из нержавеющей стали подходят друг другу.Убедитесь, что металлические поверхности чистые. Спланируйте, какой тип сварного шва вы собираетесь использовать. Пять основных типов: стык, угол, край, внахлест и т. Д. Чтобы предотвратить тепловое повреждение, зажмите кусок латуни или алюминия позади сварного шва. Он действует как теплоотвод, поэтому сталь не перегревается. Прочтите и поймите все инструкции, прилагаемые к металлу и сварочному аппарату.

Шаг 3 — Практика

Так же, как попасть в Карнеги-холл, полезно потренироваться перед большим выступлением. Так как нержавеющая сталь сложнее других металлов, даже если вы опытный сварщик, вы захотите опробовать ее на деталях, которые вам не нужны, прежде чем приступить к делу.

Шаг 4 — Сварка

Убедитесь, что металл остается на месте во время сварки. При необходимости используйте какой-нибудь зажим или приспособление. Нержавеющая сталь легко царапается, поэтому не позволяйте ей двигаться по поверхности, которая поцарапает ее. Перемещайте сварочный пистолет с постоянной скоростью, чтобы он оставался однородным. Следите за «лужей», расплавленным металлом, чтобы держать его под контролем. Будьте осторожны, когда закончите сварку, чтобы не допустить обесцвечивания стали под воздействием высоких температур.

Сварка требует контроля и навыков, поэтому лучше всего знать, что вы делаете, прежде чем начинать проект.Если вы будете делать это осторожно и заранее все спланируете, ваш проект должен хорошо выглядеть, быть полезным и сильным в течение многих лет.

Выбор электродов для сварки стержнем для начинающих: тип, размер и сила тока

Выбор электродов для электродной сварки

может быть проблематичным для начинающих и домашних сварщиков. Что нужно знать о выборе правильного типа электрода? Кроме того, как насчет правильного размера и тока электрода?

Это руководство поможет вам понять основы выбора сварочного электрода.

Содержание

Большая часть статьи посвящена электродам (или стержням) из низкоуглеродистой стали и металлам, поскольку они являются наиболее популярными.

Обзор электродов для сварки штангой

В двух словах, электроды для низкоуглеродистых и низколегированных сталей имеют обозначение, которое указывает:

  • Прочность на разрыв (Википедия) — тяговое усилие, которое металл может выдержать перед разрушением. .
  • Сварочные позиции : плоское, горизонтальное, вертикальное (вверх или вниз) и потолочное.
  • Тип флюса , покрывающий стержень:
    • Целлюлозный . Подходит для открытых корневых швов и загрязненных металлов. Стержни с таким покрытием из флюса — это E6010 и E6011.
    • Рутил . Проста в использовании и подходит для обработки тонких металлов. Стержни с таким покрытием из флюса — это E6012, E6013, E7014 и E7024.
    • Основное или с низким содержанием водорода . Для тяжелых работ. Стержни с таким покрытием из флюса — это E7016, E7018 и E7028.
  • Если в обозначении есть суффикс , это дает информацию о дополнительных легирующих элементах и ​​/ или содержании водорода в стержне.

Например, обозначение стержня E7018-1 с низким содержанием водорода:

  • 70 указывает на то, что стержень имеет минимальную прочность на разрыв 70 000 фунтов на квадратный дюйм.
  • 1 , что он сваривает во всех положениях.
  • 8 , что он имеет основное (или щелочное) флюсовое покрытие с добавлением порошка железа и низким содержанием водорода.
  • -1 , что в нем больше марганца, чем в обычном E7018.

Обозначения стержней других металлов, таких как нержавеющая сталь или чугун, указывают на химический состав.

Например, обозначение стержня из нержавеющей стали E316L-15:

  • 316 указывает количество хрома и никеля и общую коррозионную стойкость металла окончательного сварного шва.
  • L , что металл шва имеет пониженное содержание углерода для улучшения свариваемости.
  • Суффикс -15 указывает на то, что стержень имеет основное флюсовое покрытие.

Уже есть статья Weldpundit о электродных электродах для стержневой сварки, чтобы ознакомиться с ними.


Электроды для стержневой сварки

На что следует обратить внимание при выборе типа стержня для стержневой сварки?

Вы учитываете множество факторов при выборе лучшего сварочного стержня для выполняемой работы. Некоторые факторы легко выяснить, например положение при сварке. Однако некоторые другие, например, не идентифицируют тип металла.

Давайте начнем с вашего опыта и любых ограничений, касающихся вашего сварочного аппарата.

1. Ваш опыт в сварке

Если вы новичок в сварке электродом (SMAW), для ваших первых тренировочных швов и первых сварочных работ будет более полезным выбрать рутиловые стержни, например, E6013 и E7014.Эти удилища прощают много ошибок по сравнению с другими удилищами.

Когда вы станете более уверенными, вы сможете сваривать стержнями из целлюлозы, например, E6011, и стержнями с низким содержанием водорода, например, E7018. С ними труднее сваривать, но они более полезны.

2. Тип тока, который выдает ваш сварщик

Большинство аппаратов для ручной сварки вырабатывают переменный ток (AC) или постоянный ток (DC). Некоторые сварщики могут предоставить и то, и другое. В соответствии с этим у вас могут быть ограничения на удочки, которые вы можете использовать.

Если у вас есть сварочный аппарат, вырабатывающий переменный ток, вы можете использовать самые популярные низкоуглеродистые стержни, кроме E6010.

Если у вас есть современный инверторный источник питания постоянного тока, вы все равно не сможете использовать стержни E6010. Старые сварочные аппараты постоянного тока, которые больше и тяжелее, могут без проблем сжечь E6010.

Некоторые популярные стержни для нержавеющей стали, алюминия, чугуна или наплавки работают только с постоянным током.

Постоянный ток всегда дает лучшие результаты, чем переменный ток. За исключением случаев сварки намагниченных металлов.

3. Сколько напряжения холостого хода (OCV) может выдать ваш сварщик.

OCV — это напряжение между стержнем и рабочим зажимом (или зажимом заземления) до возникновения дуги.Проверьте обратную сторону сварщика или руководство, чтобы узнать, сколько OCV он предлагает.

Доступные сварочные аппараты мощностью не более 50В. Это напряжение низкое, и вы можете использовать только рутиловые стержни. Если OCV больше 70В, можно сжечь основные и целлюлозные стержни.

Кроме того, для работы большинства стержней из нержавеющей стали, чугуна и т. Д. Требуется высокий OCV.

4. Диапазон силы тока, который может выдать ваш сварщик

Еще одним соображением при выборе стержней является диапазон силы тока вашего сварщика, который должен быть в состоянии покрыть диапазон силы тока стержня.

Диапазон силы тока стержня зависит не только от диаметра стержня, но и от типа флюса.

Например, со сварочным аппаратом, который выдает 50-200 ампер, вы не можете использовать стержень E6013 1/16 дюйма (1,6 мм), который имеет диапазон 20-40 ампер. Ни 3/16 ″ (4,8 мм) стержня E7018, который имеет диапазон 200-275 ампер.

5. Какой металл вы будете сваривать

Тип металла, который вы хотите сваривать, является наиболее важным фактором при выборе стержня. Если вы не знаете, с каким типом металла имеете дело, Weldpundit предлагает подробную статью об идентификации металла.

Определить металл непросто, если вы новичок. Если вы сделаете ошибку и используете неправильный стержень, у вас будут дефекты сварки и сварной шов, который может сломаться в будущем.

Если у вас есть сомнения, вам следует обратиться за помощью к опытному сварщику, другому слесарю или поставщику для идентификации.

Однако, если вам нужно сварить незнакомые металлы, для этой работы есть стержни, которые будут описаны позже в статье.

Основными типами металлов, которые можно сваривать клеем, являются:

  • Низкоуглеродистая сталь (или низкоуглеродистая сталь).Этот металл на сегодняшний день является наиболее распространенной, доступной и свариваемой сталью. Чтобы выбрать стержень для низкоуглеродистой стали, вам необходимо соответствовать имеющемуся у него пределу прочности на разрыв. Можно использовать все обычные низкоуглеродистые стержни, например E6011, E6013 и E7014. Но вы также можете использовать обычные стержни с низким содержанием водорода, например, E7018.
  • Нержавеющая сталь. Выбирая стержень для нержавеющей стали, вы выбираете его в соответствии с его химическим составом. Прочность на растяжение вторична и не входит в обозначение стержня.
  • Углеродистые стали с содержанием углерода более 0,40% . Здесь вы выбираете стержень по прочности на разрыв, но вы используете только сухие стержни с низким содержанием водорода. Если вы используете обычные стержни или стержни с низким содержанием водорода, которые подвергаются воздействию влаги, захваченный водород приведет к растрескиванию сварного шва, если он подвергнется напряжению.
  • Стали низколегированные. Эти металлы являются низкоуглеродистыми сталями, но с дополнительными легирующими элементами. Для низколегированных сталей вы должны согласовать предел прочности на разрыв и условия эксплуатации каждого типа.Стержни из низколегированного сплава представляют собой стержни с низким содержанием водорода с необходимыми легирующими элементами во флюсовом материале. Например, E7018-A1 имеет молибден для высокой термостойкости.
  • Чугун. С чугуном все усложняется, ведь нужно правильно идентифицировать каждый тип и учитывать условия его эксплуатации. Вы можете сваривать чугун стержнями, которые могут относиться к разным категориям. Например, чугунные стержни или стержни для никелевых сплавов.
  • Алюминий. Этот металл плохо поддается сварке прилипанием даже для очень опытных сварщиков.Если вы хотите получить стержень для алюминия, он должен соответствовать серии, к которой относится заготовка.

6. Прочность металла на растяжение

При выборе стержней для углеродистой стали самое важное, что нужно сделать правильно, — это обеспечить соответствие прочности металла на разрыв. В стержнях из углеродистой стали указывается предел прочности при растяжении с помощью первых двух, а иногда и трех цифр в обозначении.

Наиболее распространенными металлами являются мягкие стали с пределом прочности на разрыв около 60 000 фунтов на квадратный дюйм (или 60 фунтов на квадратный дюйм) в зависимости от марки.

В большинстве сварочных стержней упоминается, что они имеют предел прочности на разрыв 60 фунтов на квадратный дюйм, но они имеют гораздо больше, около 10% или даже больше. В результате они могут охватывать большинство марок низкоуглеродистой стали, поэтому вам не о чем беспокоиться.

Однако некоторые марки мягкой стали могут иметь более высокий предел прочности на разрыв, который не могут покрыть стержни E 60 XX. Это обычное дело для холоднокатаной низкоуглеродистой стали. Для этих металлов вы выбираете стержни с пределом прочности на разрыв 70 фунтов на квадратный дюйм (в действительности около 80 фунтов на квадратный дюйм).

7. Назначение сварной конструкции

Если вы хотите сварить дома, которые будут выдерживать статические веса, например, сварочные тележки, столы или полки, то рутиловых стержней более чем достаточно.Даже если металлы толстые и тяжелые.

Если сварная деталь работает в более сложных условиях. Например, он подвергается воздействию низких температур или сильных ударов, для его сварки нужны стержни с низким содержанием водорода.

Наплавленный металл стержней с низким содержанием водорода обладает более высокими механическими свойствами, например пластичностью. Пластичность — это то, на сколько металл сварного шва может растягиваться без разрушения.

Особые условия эксплуатации

Если сварная деталь работает в очень тяжелых условиях, вам понадобится стержень с низким содержанием водорода и легирующими элементами, соответствующими этим условиям.

Например, вы хотите сваривать оборудование, подвергающееся воздействию очень низких температур, например, на открытом воздухе при -40 ° F (-40 ° C). Пруток E7018-1 с низким содержанием водорода позволяет наносить металл сварного шва, подходящий для таких условий.

Если вы используете целлюлозные или, что еще хуже, рутиловые стержни, металл сварного шва станет хрупким и сломается при ударе.

8. Положение для сварки

Если вы хотите сваривать в вертикальном или верхнем положении, вам понадобится стержень с цифрой «1» в обозначении, например, E70 1 8.

Чаще всего стержневые стержни свариваются во всех положениях, но большинство из них не подходят для сварки вертикально вниз, даже если они указывают на сварку во всех положениях.

Все основные стержни не подходят для вертикальной сварки вниз. Рутиловые прутки толщиной 1/8 дюйма и более тонкие можно сваривать вертикально вниз, а более толстые — нет. Чтобы быть уверенным, проверьте их упаковку, позволяют ли они вертикально вниз.

Если обозначение стержня имеет цифру «2», например, E70 2 4, вы можете выполнять сварку в горизонтальном положении. Эти стержни можно использовать для угловых швов (2F) в горизонтальном положении, но не для сварных швов с разделкой кромок (2G).

Если вам нужны настоящие универсальные удилища, способные вертикально спускаться вниз, выбирайте целлюлозные стержни.

9. Состояние поверхности заготовки

Если на заготовке (или основном металле) есть покрытия, например оцинкованная сталь, краска, ржавчина или другие формы грязи, вы должны удалить ее.

Однако, если вам нужно сваривать металлы, не очищая их, вы должны использовать стержни с самой сильной дугой. Целлюлозные стержни создают самую сильную дугу. Эти стержни могут эффективно сжигать покрытия и проникать в корродированный металл.

Рутиловые или более простые стержни имеют проблемы с покрытыми или грязными поверхностями и требуют чистого металла для хорошей работы.

10. Толщина заготовки

Рутиловые стержни больше подходят для сварки тонких металлов, например, менее 1/8 дюйма (3,2 мм), потому что они создают мягкую дугу, которая не проникает глубоко. Таким образом вы предотвратите продувание металла.

Низкоуглеродистая сталь толщиной более 0,75 дюйма (19 мм) больше не является низкоуглеродистой сталью. Для его сварки всегда следует использовать сухие стержни с низким содержанием водорода.

11. Подгонка стыка

Если вы свариваете стыки с плотной стыковкой, например квадратные стыки без корневого зазора или фаски, лучше подходят целлюлозные стержни. Эти стержни обеспечивают на 60-70% более глубокое проникновение, чем рутиловые или основные стержни.

Используйте рутиловые стержни, если стык имеет большой или неравномерный зазор. Вместо того, чтобы прожигать металл, их мягкая дуга закроет зазор.

Базовые стержни можно использовать, если заготовка большая и тяжелая, со сложным или ограниченным соединением.Основные стержни создают пластичные сварные швы, которые могут выдерживать большие нагрузки.

12. Ожидаемая деформация

Сварка выделяет много тепла и вызывает деформацию, приводящую к деформации детали. При сварке тонких металлов, например, листового металла, очень трудно избежать деформации.

Для таких ситуаций будет лучше, если у вас будет удочка не с сильной дугой, а с очень гладкой. Это означает рутиловые стержни вместо целлюлозных или основных стержней.

Рутиловые стержни также могут работать с DC-.Этот тип тока фокусирует тепло на стержне, а не на заготовке. Это поможет уменьшить искажения.

13. Внешний вид сварного шва

Если внешний вид сварного шва важен, то вот низкоуглеродистые стержни от лучшего к худшему.

  • Стержни быстрого наполнения, например E7024 и E7028. Эти стержни создают очень толстое шлаковое покрытие, препятствующее быстрому охлаждению расплавленного металла. Это самые красивые бусины. Кроме того, шлак легко удалить без особых усилий.
  • Fill-freeze стержней, например, E7018, E7014 и E6013. Их шлаковое покрытие достаточно толстое, чтобы обеспечить красивый внешний вид, и его легко удалить без особых усилий.
  • Быстрозамороженные стержни , например, E6010 и E6011. Эти стержни создают на валике тонкий шлак. Из-за этого расплавленный металл замерзает, прежде чем сможет получить гладкую поверхность. Кроме того, шлак не снимается без повторного использования отбойного молотка, что ухудшает внешний вид борта.

14. Производительность и количество сварочных проходов

Сварка палкой — медленный процесс. Если у вас много работы, стержни с добавлением железа во флюсе могут ускорить процесс. Например, вместо E6013 вы можете выбрать E7014 или, что еще лучше, E7024.

Если вы хотите выполнить многопроходную сварку, целлюлозные стержни могут замедлить работу. Они производят шлак, который нелегко удалить и который требует большой очистки между проходами. Кроме того, они не содержат большого количества железного порошка во флюсе.

Для этих сварных швов лучше всего использовать стержни из целлюлозы только для корневого прохода. После этого используйте другие стержни с высоким содержанием железного порошка для заполнения стыка.

15. Стоимость сварки

Вы всегда должны выбирать стержень, который удовлетворяет требованиям проекта, но при этом является наиболее эффективным с точки зрения затрат и времени.

Например, если вы используете низкоуглеродистую сталь для сварки простой конструкции, вы можете использовать простой E6013 вместо E7018 с низким содержанием водорода. Да, E7018 сильнее, но дополнительные силы тратятся на такую ​​работу, она стоит немного дороже, и новичку будет сложно запустить и перезапустить дугу.

Другой пример — использование прутка E316 для сварки заготовки из нержавеющей стали 304. Стержень из стали 316 обеспечивает превосходную коррозионную стойкость, которая теряется на заготовке из 304. Удилища 308 более чем достаточно, его легче найти и он стоит дешевле.

Кроме того, стержни с тонкими диаметрами 1/16 ″ и 5/64 ″ дороже, чем стержни обычного диаметра.

Штанги быстрой заливки, такие как E7024, обеспечивают высокую производительность и могут сэкономить много времени, если вам нужно много работать с более толстыми металлами.


Какие штанги используются чаще всего для домашней сварки?

Наиболее распространенными стержнями для стержневой сварки являются E6011, E6013, E7024 и E7018 для углеродистой стали.Эти стержни могут удовлетворить почти все потребности домашнего сварщика для низкоуглеродистой стали.

Теперь давайте посмотрим, как в основном используются эти стержни для начинающих и домашних сварщиков.

E6011 для глубокого проплавления и грязных металлов

Штанга E6011 используется для сварки низкоуглеродистых сталей, для которых требуется дуга с глубоким проплавлением, когда металл не является чистым и металл шва должен быстро замерзнуть.

Пруток E6011 часто выбирают домашние сварщики и идеально подходят для сварки:

  • Открытые корневые швы.
  • Соединения с плотной посадкой.
  • Вертикально-нижнее положение.
  • Грязный металл.
  • Быстрое и надежное обслуживание уличного оборудования.

Однако сильная дуга затрудняет использование E6011 для начинающих. Кроме того, быстро замораживающийся металл сварного шва не дает хорошего вида валикам.

Есть также штанга E6010, похожая, но лучше, чем E6011. Однако, как уже упоминалось, большинство домашних сварочных аппаратов не могут с ним работать.

E6013 для наиболее распространенных производств

Штанга E6013 используется для сварки низкоуглеродистой стали для некритических работ с очень удовлетворительными результатами.E6013 идеально подходит для сваривания самых разнообразных предметов домашнего обихода.

Стержни E6013 популярны, потому что:

  • Они очень просты в использовании.
  • Легко найти.
  • Все сварочные аппараты могут их сжечь, потому что им для работы требуется низкий OCV.
  • Они поддерживают как постоянный, так и переменный ток.
  • Может сваривать очень тонкий металл.
  • Они производят красивый бисер.
  • Их легко хранить, достаточно беречь от излишней влаги.

Однако штанги E6013 не обеспечивают хорошей видимости лужи и склонны к включению шлака.

E7024 для более быстрой обработки больших стыков

Стержень E7024 используется для наплавки большого количества металла шва на толстую низкоуглеродистую сталь в плоском положении. Их высокое осаждение металла экономит много времени.

E7024 представляет собой рутиловый стержень, аналогичный E6013 и E7014, но он содержит больше порошка железа во флюсе. Не менее 50% флюса составляет железный порошок.

Добавленное железо делает E7024 быстро заполняющимся стержнем и может осаждать большое количество сварочного металла в стыке. Кроме того, он предлагает самые красивые бусинки и прост в использовании.

Однако этот стержень нельзя использовать для вертикальных, потолочных или горизонтальных стыковых швов. Кроме того, он не подходит для тонкого металла, например, тоньше 3/16 дюйма (4,8 мм).

Наконец, вам нужен сварочный аппарат, который может выдавать большую силу тока для работы со стержнями E7024.

E7018 для более трудно свариваемых металлов и соединений

Штанга E7018 используется для сварки низкоуглеродистых, низколегированных и некоторых трудно свариваемых сталей. E7018 также сваривает соединения, которые должны выдерживать нагрузки, которые другие типы стержней не могут выдержать.

Сварочные стержни E7018 — это стержни с основным (или с низким содержанием водорода) стержнем, которые подходят для тяжелых работ. Их можно использовать в случаях, когда сваривать трудно, например, в следующих случаях:

  • Сталь с более высоким содержанием углерода, чем низкоуглеродистая сталь.
  • Стали простые низколегированные. За исключением случаев, когда для их состава требуется более подходящий стержень.
  • Низкокачественная сталь с высоким содержанием серы. Основной флюсовый материал E7018 может справиться с этим.
  • Сварные детали, которые подвергаются продолжительным или сильным ударам.
  • Сварные швы, которые должны выдерживать удары при очень низких температурах.
  • Большие стыки сложной конструкции или узкие стыки, которые могут треснуть, если их сварить целлюлозными или рутиловыми стержнями.

Основной флюс стержня E7018 защищает металл шва от неметаллических включений, вызванных кислородом. Эти включения ослабляют металл шва. Базовый флюс также борется с примесями в стали низкого качества, такими как сера, которые могут привести к горячему растрескиванию.

Полученный металл шва обладает высокой пластичностью, способной выдерживать сильные остаточные напряжения, сильные удары, низкие температуры и т. Д.

Ручная сварка с низким содержанием водорода

Уникальное преимущество E7018 заключается в том, что он содержит низкий уровень водорода. Водород внутри зоны термического влияния (HAZ) закаленной стали может вызвать растрескивание, если подвергнется сильному напряжению. HAZ — это обесцвеченная область рядом с буртиком.

Низкоуглеродистая сталь не может стать достаточно твердой для водородного растрескивания, за исключением случаев, когда ее толщина превышает 0,75 дюйма (19 мм). Все другие углеродистые стали с повышенным содержанием углерода или легирующих элементов могут стать твердыми при сварке и требуют стержней с низким содержанием водорода.

Однако сварка с низким содержанием водорода — это строгий процесс. Вы должны использовать стержни с низким содержанием водорода в течение 4 часов с момента открытия упаковки. После этого необходимо их повторно просушить в специальных печах для сварки прутков. Эти печи очень дороги.

Для сварки с низким содержанием водорода также необходимы правильный предварительный и последующий нагрев основного металла, безупречная техника сварки и т. Д. Одним словом, сварка с низким содержанием водорода — это профессиональная работа.

Вы можете использовать стержни E7018 дома для более прочных, чем обычные сварные швы общего назначения, но не для критически важных работ с низким содержанием водорода.Критическая работа — это когда суставы ломаются, и результатом являются травмы и повреждение тяжелого оборудования, например, ремонт прицепов и строительных конструкций.

Сравнительная таблица самых популярных сварочных электродов для стержневой сварки

Вот сравнительная таблица самых известных сварочных электродов для низкоуглеродистой стали, чтобы получить быстрый обзор.

все 909 904 6
E6011 E6013 E7014 E7024 E7018
Блок питания
совместимость
4 9047
совместимость
4 6 10 10 10 7
Проникновение 10 5 6 5 7
9047 9047 9047 9047 9047 8
Грязные металлы 9 4 5 4 3
Листовой металл 7 10-
Тонкий металл 7 9 8 6 4
Th ick metal 8 8 8 8 10
Твердосвариваемые металлы 5 3 3 4 8 9 10 8
Скорость осаждения 4 5 6 10 6
7
Простое хранение 9 7 7 7 2
Таблица сравнения самых популярных электродов для стержневой сварки

Как выбрать правильный размер стержня для стержневой сварки?

После выбора правильного типа стержня вы выбираете правильный размер.

Правильный размер стержня для стержневой сварки — это наибольший размер, который подходит для соединения, чтобы обеспечить надлежащее проплавление и сплавление в сочетании с большим наплавлением металла. Выбрав максимально возможный размер стержня, вы будете сваривать быстро, избегая при этом слишком большого нагрева стыка.

Если размер стержня слишком мал, он может не обеспечить глубокого проникновения и сплавления с основным металлом. Включения шлака, которые застревают в металле шва, чаще встречаются в стержнях, которые слишком тонкие для работы.

Для заполнения стыка потребуется сварить дополнительные проходы. Помимо того, что на это нужно тратить время, основной металл может деформироваться из-за чрезмерного тепла, которое попадает в стык. Будет лучше, если вы будете ждать между каждым проходом, чтобы температура снизилась.

Если стержень слишком толстый для соединения, дуга не достигнет основания. Вместо этого он прыгнет на ближайший металл.

В результате проникновение корней уменьшается или вообще отсутствует. Также под металлом шва могли быть включения шлака.

Общая толщина стержня и диаметр проволоки

Диаметр стержня — это диаметр проволоки без покрытия из флюса. Толщина флюса зависит от количества шлака, который образует флюс, и добавленного порошка железа.

Прутки разных типов могут иметь одинаковый диаметр проволоки, но разную общую толщину.

Электроды для ручной сварки с одинаковым диаметром проволоки, но разной общей толщиной

Стержни для быстрой заливки имеют большое отношение флюса к диаметру проволоки, затем стержни для заливки и замораживания являются средними, и, наконец, стержни для быстрой заморозки имеют тонкий флюс по сравнению к проводу.

Например, стержень E7014 1/8 дюйма толще, чем E6013 1/8 дюйма, и намного толще, чем E6010 1/8 дюйма. Но он тоньше стержня 1/8 ″ E7024. Каждый по-разному подходит к одному и тому же стыку.

Толщина основного металла

Как правило, для односторонних однопроходных стыковых швов толщиной металла до 3/16 ″ вы выбираете сварочный стержень, который на ступеньку тоньше основного металла. Например, если основной металл составляет 1/8 дюйма, вы выбираете стержень 3/32 дюйма.

Однако общее правило гибкое.Вы можете выбрать стержень, который примерно вдвое меньше основного металла, и это даст хорошие результаты. Если стержень тоньше этого, вы можете сваривать, но скорость наплавки металла неудовлетворительна.

Вы можете выбрать стержень той же толщины, что и основной металл, но используйте его с осторожностью. Тепло, которое проникает в основной металл, может продуть металл и вызвать чрезмерное коробление.

Неправильно выбирать стержень толще основного металла. В этой ситуации тепло, которое попадает в основной металл, слишком велико, что делает сварку невозможной для начинающих.

Единственным исключением из этого правила является сварка очень тонких металлов, например, листового металла 17 калибра (1,4 мм), в то время как самый тонкий стержень составляет 1/16 дюйма (1,6 мм).

Но сварка палкой очень тонких металлов — дело жесткое и требует большого опыта, но дает плохие результаты.

Если основной металл толще 3/16 ″ и для достижения полного проплавления при односторонней сварке, можно скосить стык, например, создать одно V-образное стыковое соединение. Затем вы свариваете стык, используя стержни обычного размера.Пример следует позже.

Положение при сварке

Положение при сварке также важно при выборе размера стержня.

При сварке в вертикальном или верхнем положении вы выбираете меньший размер стержня, чем в плоском положении. Для вертикального опускания вы выбираете такой же, а иногда и более толстый размер удилища, чем в горизонтальном положении.

Меньшие стержни образуют лужу меньшего размера, которую легче контролировать.

Примеры выбора размера стержня стержня

Вот два примера выбора размера стержня стержня для стыковых швов.Размеры стержней указаны для стержней для замораживания наполнения, таких как E7018 и E6013.

Квадратное стыковое соединение с открытым основанием

Допустим, вы хотите сварить квадратное стыковое соединение в плоском положении, при этом толщина основного металла составляет 1/8 дюйма (3,2 мм). Вы можете оставить корневую щель размером в половину толщины металла 1/16 дюйма (1,6 мм), но не более. Затем для сварки используйте пруток 3/32 дюйма (2,4 мм).

Стыковое соединение с одним V-образным вырезом

Если вы хотите сварить металл толщиной 1/4 дюйма (6,4 мм), вы сделаете фаски под углом 30 градусов. Затем оставьте корневую поверхность (или землю) длиной 3/32 дюйма и корневую щель длиной 1/16 дюйма.Это одинарный V-образный стык.

Вы привариваете корень стержнем 3/32 дюйма. После этого вы можете заполнить оставшуюся часть стыка стержнями 1/8 ″ (3,2 мм) или 5/32 ″ (4 мм).

Если вы используете целлюлозные стержни, всегда рекомендуется оставлять тонкий зазор между корнями, как в предыдущем примере, поскольку они проникают больше. С другими стержнями вы можете оставить корневой зазор, равный поверхности корня, но не более.

Каковы наиболее распространенные размеры стержней?

Наиболее распространенные размеры стержней для стержневой сварки — 3/32 дюйма (2.4 мм), 1/8 дюйма (3,2 мм) и 5/32 дюйма (4 мм). Этих размеров достаточно, чтобы сваривать наиболее распространенные объекты сваркой штучной сваркой.

Стержни малого диаметра, например, 1/16 ″ (1,6 мм) и 5/64 ″ (2 мм), сваривать сложнее, чем стержни обычного размера.

Это связано с тем, что вы используете эти диаметры для деталей толщиной менее 1/8 дюйма (3,2 мм), что является наименьшим рекомендуемым размером для сварки с помощью процесса сварки штангой.

Кроме того, стержни малого диаметра имеют тенденцию к вибрации на конце и требуют очень устойчивых рук.Наконец, их труднее найти и они более дорогие.

Штанги стержней 3/16 ″ и больше используются для очень толстых и больших заготовок, которые редко встречаются и требуют большого опыта для сварки. Они образуют большую лужу, которую трудно контролировать.

Кроме того, для больших стержней вам понадобится сварочный аппарат с большой силой тока. Возможно трехфазное электроснабжение.

Наконец, с увеличением диаметра стержня опасность сварки также возрастает. Вы будете иметь дело с большим количеством тепла, ультрафиолетовых лучей, сварочного дыма, брызг, шума и т. Д.


Как выбрать силу тока для стержня для стержневой сварки?

После того, как вы выбрали правильный тип и размер, вы выбираете правильную силу тока для стержня ручки.

Вы выбираете силу тока стержня для стержневой сварки, устанавливая ее достаточно высоко, чтобы обеспечить наилучшее проплавление и сварку для выполняемой работы, всего за один шаг до того, как слишком большое количество тепла приведет к дефектам.

При сварке вам нужно лучшее проплавление и сплавление металлов, с которыми вы работаете. Для достижения этих результатов вы устанавливаете высокую силу тока, чтобы выделять достаточно тепла, чтобы расплавить стержень, металлы и смешать их.

Тип материала флюса влияет на силу тока, необходимую для плавления стержня, в первую очередь, от дополнительного порошка железа. В сочетании с диаметром стержня существует диапазон силы тока, в котором стержень может работать эффективно.

Например, E7018 3/32 дюйма имеет диапазон силы тока 70–110 А, а E6011 3/32 дюйма — диапазон силы тока 60-90 А.

Если вы установите силу тока ниже указанного диапазона, дуга будет только искрой, а стержень будет прилипать к металлу.

Если сила тока выше допустимого диапазона, дуга будет действовать и звучать слишком агрессивно, флюс будет поврежден до того, как он загорится, и стержень станет красным во время сварки.

Какая точка отсчета силы тока хороша?

Поскольку вы выбрали правильный тип и размер стержня для основного металла, давайте посмотрим, что может стать хорошей отправной точкой при выборе силы тока для обычных сварных швов.

Предположим, вы привариваете угловой элемент в горизонтальном положении (2F), и установите силу тока равной среднему диапазону силы тока стержня. Если вы выполняете стыковой шов в плоском положении (1G), вы устанавливаете силу тока несколько ниже.

Например, 1/8 ″ E7014 имеет диапазон силы тока 100–150.Идеальная начальная сила тока этого стержня для сварного шва 2F составляет 125 А.

Эта сила тока предохранит стержень от прилипания, скорость плавления будет высокой, а тепла будет более чем достаточно для хорошего плавления. При этом побочных эффектов от слишком большого количества тепла не будет.

Следуя этому общему правилу, вы точно настраиваете силу тока с учетом следующих факторов.

Размер основного металла

Тонкие и маленькие заготовки не переносят большого количества тепла, потому что оно пропитывает металл и приводит к деформации.Даже если размер стержня правильно входит в соединение, вы хотите снизить силу тока.

Если у вас очень большая и толстая заготовка, она будет поглощать тепло с большой скоростью. Это приведет к плохому проникновению и включению шлака. По этим причинам вы устанавливаете большую силу тока.

Температура основного металла

Если температура основного металла очень низкая, это похоже на сварку с меньшей силой тока. Дуга не зажжется легко, и стержень будет прилипать к основному металлу.Чтобы этого не произошло, установите силу тока немного выше или предварительно нагрейте основной металл.

Если температура основного металла слишком высока по сравнению с предыдущей сваркой, это похоже на сварку с большей силой тока. Основной металл перегреется и станет красным, особенно когда вы дойдете до конца соединения. Чтобы этого избежать, уменьшите силу тока.

Положение при сварке

Как и в случае выбора размера стержня, положение при сварке очень важно при установке силы тока. Вы используете разную силу тока для разных положений сварки при одинаковом размере стержня.

Следует обратить внимание на то, что при точной настройке силы тока вы не получаете больше или меньше тепла. Вы пытаетесь добиться того же тепла, что и в плоском положении.

Когда вы выполняете сварку в положении на потолке , тепло увеличивается и концентрируется на металле. Это похоже на сварку с большей силой тока. Лужа станет слишком жидкой, и она не останется в стыке. В верхнем положении вы уменьшаете силу тока примерно на 5%.

В положении вертикально вверх тепло распространяется к верхнему концу основного металла во время сварки.Верх перегреется, покраснеет и, наконец, растает. Чтобы предотвратить это, вы уменьшите силу тока примерно на 10% или более при сварке в вертикальном верхнем положении.

При сварке вертикально вниз вы устанавливаете силу тока выше примерно на 10-15%. Это потому, что вам нужна высокая скорость сварки, а стержень должен плавиться достаточно быстро.

Тип соединения и подгонка

При сварке угловых швов требуется более высокая сила тока, чем при сварке стыковых швов.

В зависимости от корневого зазора и размеров торца, а также размера стержня сила тока может быть на 10-20% ниже, чем обычно для сварных швов с открытым корнем.Таким образом вы избежите продувания металла. Любые дополнительные проходы требуют типичной силы тока для стержня.

Для соединений с плотной подгонкой требуется более высокая сила тока, а для соединений с очень широкой или неравномерной установкой — меньшая сила тока.

Тип тока

При сварке на переменном токе вы устанавливаете силу тока немного выше, чем постоянный ток. Это потому, что переменный ток постоянно меняет направление, и между каждым циклом наблюдается перепад тепла.

Функция дуговой силы

Современные инверторные сварочные аппараты поддерживают функцию, называемую дуговой силой или копанием.Если вы установите высокое значение силы дуги, источник сварочного тока увеличит силу тока, если длина дуги укорачивается.

Сила дуги полезна, потому что она предотвратит прилипание стержня к основному металлу, если вы вставите стержень в соединение.

Однако использование силы дуги похоже на сварку с большей силой тока. Поэтому, если вы устанавливаете высокое значение силы дуги, вы должны учитывать это при выборе силы тока стержня.

Как определить, слишком ли высокий или низкий ток при сварке электродом?

Во-первых, имейте в виду, что значения силы тока в статьях, видео и диаграммах являются приблизительными, чтобы дать вам хорошее начало.

Большая проблема в том, что все сварщики по-разному отображают силу тока. Более дорогие машины надежнее.

Например, вы установили для одного сварочного аппарата 100А, но на самом деле он дает вам 85А. Вы устанавливаете еще один на 100А, и он дает вам 95А.

Кроме того, многие сварочные аппараты с двойным напряжением выдают разную силу тока в зависимости от напряжения, которое вы их подключаете.

Например, вы устанавливаете ручку управления сварочного аппарата на 150 А, но сварочный аппарат будет выдавать 120 А при подключении к 120 В и 160 А при подключении к 240 В.Ознакомьтесь с руководством сварщика или обратитесь к производителю по этому поводу.

Только надежный амперметр (Википедия) может показать вам действительную силу тока. Если вы хотите использовать его со сварочным аппаратом постоянного тока, убедитесь, что амперметр может работать с постоянным током.

Более того, все сварщики используют разное напряжение дуги для одинакового размера стержня и силы тока. Напряжение дуги не влияет на тепло напрямую, как сила тока, но напрямую влияет на температуру.

Наконец, вы можете неправильно рассчитать коэффициент выбора силы тока и в итоге получить неверную силу тока.

Вы должны сразу понять, если вы выполняете сварку с неправильной силой тока, чтобы избежать дефектов.

Обладая достаточным опытом, вы можете быть уверены в правильной силе тока для выполняемой работы.

Вы узнаете правильную силу тока по внешнему виду и поведению лужи в сочетании со звуком дуги.

При сварке с низким током
  • Вам будет труднее запустить дугу.
  • Пруток заедает во время сварки.
  • Дуга выглядит тусклой и слабой.
  • Звук дуги будет слабым и непостоянным.
  • Лужа будет узкой, маленькой и плохо промокнет.
  • Расплавленный шлак будет темнее обычного.
  • Скорость движения ниже оптимальной.
  • Последний борт будет высоким и узким с плохой сваркой.
При сварке с большой силой тока
  • Дуга будет хаотично звучать и будет выглядеть очень яркой.
  • Лужа будет слишком широкой, жидкой, и ее будет труднее контролировать.
  • Расплавленный шлак будет иметь более длинный хвост и более красный, чем обычно.
  • Скорость движения выше оптимальной.
  • У вас будет большее количество брызг.
  • Бортик будет слишком широким и плоским с небольшим подрезом.

Полезный совет для новичков — проверить силу тока на аналогичном металлоломе, чтобы определить правильную силу тока. После этого можно без дефектов сваривать заготовку.

Таблица с диапазоном силы тока популярных сварочных стержней

Полезная таблица с размерами и диапазоном силы тока популярных электродов для сварки штангой.Диапазон силы тока стержня каждого производителя немного отличается. Вы всегда должны проверять упаковку удилища, чтобы точно знать диапазон.

46-195

90 457 9046 Палка диапазон силы тока сварочных электродов на диаметр проволоки DC +

Примеры выбора силы тока и стержня стержня

Теперь несколько примеров выбора силы тока.Сила тока указана для угловых швов. Для стыкового шва с открытым корнем уменьшите силу тока, чтобы избежать продувки металла.

Для сварки заготовки толщиной 3/16 ″ со стержнем 1/8 ″ E6011 :

  • Для горизонтального использования 100A.
  • Для вертикального опускания используйте 115А и более высокие скорости.
  • Для вертикального подъема используйте более тонкий стержень 3/32 ″ и 65A.

Для сварки заготовки толщиной 3/32 ″ с помощью стержня 3/32 ″ E6013 (они имеют одинаковую толщину):

  • Для горизонтального использования 65A.
  • Для вертикального опускания используйте 75A.
  • Для вертикального подъема используйте более тонкий стержень 5/64 ″ и 40A.

Для сварки заготовки 3/16 ″ толщиной стержнем 3/32 ″ E7018 :

  • Для горизонтального использования 95A. Или вы можете использовать более толстый пруток 1/8 дюйма и ток 130 А, чтобы сваривать быстрее.
  • Для вертикального подъема используйте 80A.
  • Этот пруток не допускает сварки вертикально вниз.

В двух словах

Выбор сварочного стержня немного сложен для новичков.Вот резюме.

Стержни E6013, E6011 и, возможно, E7018 — это стержни, которые вы будете использовать для сварки низкоуглеродистой стали в качестве новичка. Убедитесь, что у вас есть сварочный аппарат, способный работать с двумя последними.

Вы всегда должны выбирать размер стержня тоньше основного металла. Убедитесь, что размер стержня соответствует размеру сустава, чтобы обеспечить глубокое проникновение в корень. Однако размер стержня также должен обеспечивать удовлетворительное осаждение металла.

Установите большую силу тока, за шаг до того, как это может вызвать побочные эффекты в виде лужи или беспорядочный звук дуги.


Другие статьи Weldpundit

Какой сварочный стержень использовать для чугуна? Полное руководство.

Какой самый простой в использовании стержневой сварочный стержень?

Что такое прокатная окалина для горячекатаной стали: и как сваривать ее.

Сварка палкой — легко или сложно научиться? и сколько времени это займет.

Какой респиратор вам нужен для домашней сварки?

Почему сварочные стержни остаются прилипшими? и что с этим делать.

Можно ли сварить дома? Основные соображения.

Можно ли приваривать нержавеющую сталь к углеродистой стали? Руководство для начинающих.

4 Популярные типы процедур сварки

Статья обновлена ​​3 июня 2021 г. и предлагает гораздо более подробную информацию о типах металлов, их использовании, методах сварки и расположении, а также о том, как учитываются состав и точки плавления различных металлов. К процедурам диаграмм добавлена ​​подробная инфографика.

Работа с металлом увлекательна и вдохновляет. По мере того как разлетаются искры и повышается тепло, сварщики могут преобразовывать одни из самых прочных материалов в мире в формы и изделия, которые они себе представляют.Этот навык требует работы и практики, и его лучше всего усвоить с помощью и руководством профессионалов отрасли.

Изучение основ новой профессии может занять много времени. Вам необходимо ознакомиться со всем рабочим процессом от начала до конца и освоить каждый уровень, прежде чем двигаться дальше. Внимание к деталям — вот что делает хорошего сварщика более разносторонним потенциальным сотрудником. Есть четыре основных типа сварочных процедур, которые студенты Lincoln Tech должны изучить, чтобы стать успешными сварщиками, работающими в этой области.Студенты Lincoln имеют уникальную возможность пройти комплексную практическую подготовку у опытных инструкторов. Под руководством одних из лучших в отрасли студенты освоят четыре самых популярных типа сварочных процедур.

4 типа сварочных процессов

Газовая дуговая сварка металла (GMAW / MIG)

Этот вид сварки также называется сваркой в ​​среде инертного газа (MIG). Он использует защитный газ вдоль проволочного электрода, который нагревает два соединяемых металла.Этот метод требует постоянного напряжения и источника питания постоянного тока и является наиболее распространенным промышленным процессом сварки, который включает в себя пластины и трубы с большим диаметром отверстия.

В процессе сварки GMAW / MIG используются четыре основных метода переноса металла:
  1. Шаровидный перенос дает более грубый сварной шов из-за размера капель металла и склонности к разбрызгиванию. Этот метод удобен для сварки толстых металлических листов в горизонтальном положении.
  2. Короткое замыкание работает, как следует из названия — сварочная проволока контактирует с основным металлом, быстро повторяясь, много раз в секунду.Поскольку в процессе сварки образуется небольшое количество брызг, этот метод можно использовать в любом положении сварки.
  3. Распылительный перенос передает крошечные капельки расплавленного металла с такой стабильной скоростью, что обеспечивает устойчивую контактную дуговую сварку во время процесса. Несмотря на то, что этот метод приводит к небольшому разбрызгиванию, его лучше всего использовать на толстых и плоских горизонтальных предметах.
  4. Импульсное распыление очень похоже на распыление, но использует импульс сильного и слабого тока для обеспечения периодов микроохлаждения.Благодаря такому типу поставки этот процесс может использоваться для металлических листов различной толщины и практически во всех положениях сварки. Обратите внимание, что когда мы используем термин «охлаждение» при описании импульсного распыления, моменты более низкого напряжения, которые обеспечивают более холодный сварной шов, по-прежнему составляют несколько тысяч градусов по Фаренгейту. Он считается более холодным по сравнению с высоковольтной частью цикла. При любой сварке используются экстремальные температуры.

Газовая дуговая сварка вольфрамом (GTAW / TIG)

Сварка вместе толстых секций нержавеющей стали или цветных металлов является наиболее распространенным применением этого метода.Это процесс дуговой сварки, в котором для создания сварного шва используется фиксированный плавящийся вольфрамовый электрод. Этот процесс занимает гораздо больше времени, чем сварка MIG, дуговой сваркой стержнем или порошковой проволокой.

Температуры плавления цветных металлов значительно различаются, поэтому необходимо соблюдать осторожность при определении состава основного металла. И нержавеющая сталь, и сталь содержат железо, однако, чтобы считаться нержавеющей сталью, металл должен содержать не менее 11% хрома. Углеродистая сталь плавится при температуре от 2600 до 2800 градусов F.

Присутствие 11% хрома в нержавеющей стали сужает этот температурный диапазон до отметки 2750 +/- градусов F. Но ничто не свидетельствует о сварочных навыках лучше, чем умение сваривать алюминий TIG. Этот навык требует твердой руки, натренированного глаза и художественного чутья, чтобы создать гладкий, потрясающий сварной шов.


Дуговая сварка экранированного металла (SMAW)

В этом конкретном типе сварки сварщик следует ручному процессу сварки штангой. Палка использует электрический ток для образования дуги между палкой и соединяемыми металлами.

Это часто используется при строительстве стальных конструкций и в промышленном производстве для сварки чугуна, стали и использования открытой V-образной канавки при сварке труб из низкоуглеродистой стали.

Жизненно важно, чтобы сварщик уметь сваривать до уровня, при котором его работа может пройти испытание на изгиб разрушающего типа. Хотя дуговая сварка защищенным металлом используется для соединения углеродистой стали, легированных сталей, нержавеющей стали, чугуна и высокопрочного чугуна, ее также можно использовать для обработки некоторых цветных металлов, таких как никель и медь.Редко используется на алюминии.


Дуговая сварка порошковой проволокой (FCAW)

Этот метод был разработан как альтернатива сварке под защитным экраном. Полуавтоматическая дуговая сварка часто используется в строительных проектах благодаря высокой скорости сварки и портативности. Этот метод имеет множество переменных, что делает его применимым в различных сварочных проектах. Переменные часто зависят от модели используемого сварочного аппарата и от того, какой тип проволоки был выбран для применения.

Гибкость увеличивается за счет множества рабочих углов, уровней напряжения, используемой полярности, а также скорости подачи проволоки.Из-за возможности более высоких скоростей сварки вновь соединенный металл остывает быстрее. Если сварщик использует порошковую проволоку, он или она должны следить за пористостью сварного соединения.

Дуговую сварку порошковой проволокой лучше всего использовать на открытом воздухе или под вытяжками промышленной вентиляции из-за большого количества дыма и дыма, образующихся в процессе сварки.


Обучение и трудоустройство в отрасли

Вышеупомянутые четыре типа сварки обычно используются в большинстве промышленных и строительных приложений и обеспечивают множество полезных и востребованных навыков.Каждый требует значительной практики и знаний. Программа Lincoln Tech Welding Technology обучает выпускников необходимым навыкам и знаниям, чтобы начать карьеру в этой области.

Узнайте, как сваривать в девяти центрах Lincoln Tech

Если вы готовы узнать больше о сварочной отрасли и подумываете о том, чтобы стать сварщиком, посетите одну из девяти школ сварки Lincoln Tech, расположенных в Восточном Виндзоре, штат Коннектикут; Денвер, Колорадо; Гранд-Прери, Техас; Индианаполис, Индиана; Колумбия, Мэриленд; Саут-Плейнфилд, штат Нью-Джерси; Мелроуз, Иллинойс; Нэшвилл, Теннесси; и Махва, штат Нью-Джерси.

Недостаток навыков дает возможность трудоустройства

Обладая большим опытом в этих сварочных технологиях, выпускники могут увидеть много возможностей, открывающихся перед ними, когда они начнут искать работу. Сварщики часто ищут работу на производстве, в коммерческом строительстве, горнодобывающей промышленности, сельском хозяйстве, оптовой торговле, художественных ограждениях, а также в ремонте и обслуживании оборудования. Разнообразие применения этого навыка открывает прекрасные возможности для трудоустройства.

Как выбрать подходящего сварочного аппарата для ваших нужд (MIG, Stick и TIG)

Если вы новичок в сварке, широкий ассортимент продукции на рынке на первый взгляд может показаться умопомрачительным.

Подобно Ford, Toyota и Mercedes Benz в автомобильной промышленности, существует несколько крупных производителей сварочных аппаратов. Большие мальчики — это Линкольн, Миллер, Хобарт (сейчас принадлежит Миллеру) и ЭСАБ.

Так же, как автопроизводители, выпускающие седаны, пикапы, спортивные купе и внедорожники, существует несколько «моделей» сварочных аппаратов, каждая из которых служит разным целям и обслуживает определенных пользователей.

Выбрать подходящий вам не составит труда. Приведенная ниже информация поможет вам в этом процессе.

Виды сварщиков

Наиболее часто используемые сварочные аппараты:

  • металлический инертный газ (MIG)
  • вольфрамовый инертный газ (TIG)
  • дуговая сварка металлическим электродом в защитных оболочках (SMAW или Stick)
  • Аппараты для кислородно-ацетиленовой сварки («газовые» или «кислородные»)

Существуют также универсальные и более дорогие многопроцессорные машины, которые могут сваривать более чем одним процессом. Также есть сварочные аппараты с моторным приводом (на топливе) для работы от электросети.(Эти более сложные сварщики будут описаны в разных статьях.)

В рамках этого введения мы сосредоточимся на основных аппаратах для сварки MIG, TIG и стержневой сваркой. Если вы не знакомы с различными сварочными процессами, ознакомьтесь с основным руководством по сварочным процессам, прежде чем продолжить здесь.

Понимание основных процессов важно для нового или начинающего сварщика. Ваши перспективы трудоустройства резко возрастут, если вы поймете особенности и преимущества, связанные с различными типами сварщиков.

Например, знание того, какая модель лучше всего подходит для конкретного задания и какой присадочный пруток, проволока или стержневой электрод лучше всего соответствует требованиям норм, дает вам право на работу в качестве руководителя, помощника по проекту, техника по сварке или покупателя в вашей компании.

Эта статья начинается с основ выбора машины. После этого я покажу вам, как читать «спецификации», включенные в литературу по продажам продуктов, и научу сравнивать ключевые характеристики.

Шаг 1. Определите тип металла, который вы будете сваривать

Углеродистая сталь

Большинство сварных швов выполняются на трубах из углеродистой стали или листовом металле.Углеродистая сталь (или обычная сталь) может выдерживать много тепла. Таким образом, в отличие от других металлов, перечисленных ниже, этот металл очень прощает, когда начинающий сварщик применяет слишком много тепла.

В большинстве сварочных процессов используется углеродистая сталь. Кроме того, для получения красивого сварного шва на станке не нужно много дополнительных функций.

Нержавеющая сталь

Нержавеющая сталь (SS) более требовательна к теплу. Изготовители состоят из стали, хрома и никеля и используют этот сплав для изготовления емкостей для пищевых продуктов / напитков и многих других продуктов, в основном из-за его антикоррозионных свойств.

Рабочие обычно сваривают нержавеющую сталь, используя аппараты MIG или TIG. Сварные швы из нержавеющей стали требуют меньшего тока, чем углеродистая сталь, для решения проблемы более низкой теплопроводности металла. Кроме того, повышенная сложность большинства сварных швов из нержавеющей стали требует, чтобы сварщик имел приличный контроль, чтобы получить правильную дугу и / или лужу.

Также можно найти стержневые электроды из нержавеющей стали. Это позволяет использовать сварочный аппарат для выполнения работы. Но имейте в виду, что основной металл должен быть достаточно толстым, чтобы выдерживать высокую температуру аппарата для ручной сварки, и вы часто получаете больше брызг по сравнению с аппаратом для сварки MIG или TIG.

Алюминий

Для сварщиков алюминий приходит с другой планеты.

Как цветной металл, алюминий настолько хорошо проводит тепло, что вам постоянно требуется его больше, чтобы лужа оставалась расплавленной. В то же время заготовка легко деформируется, если становится слишком горячей. Следовательно, алюминий часто требует более сложного оборудования для выполнения своей работы.

Для алюминия можно использовать сварочные аппараты MIG (особенно с функцией импульсной сварки). Тем не менее, многие механизмы подачи проволоки испытывают трудности с подачей алюминиевой присадочной проволоки.Поэтому необходимо приобрести отдельную надстройку, называемую катушкодержателем.

Если вы собираетесь работать с алюминием, убедитесь, что ваш сварочный аппарат MIG может работать с пистолетом для катушки. Не все из них.

Сварочный аппарат TIG, разработанный для алюминия, должен иметь выход переменного тока (AC-TIG). Другие полезные функции включают инверторную электронику, технологию прямоугольной волны, управление балансом и импульсную опцию. Естественно, эти дополнительные функции повысят стоимость сварщика TIG.

Хотя это не лучший выбор, аппарат для ручной сварки также может сваривать алюминий.Как и нержавеющая сталь, основной металл должен быть достаточно толстым, чтобы выдерживать высокие температуры.

Титан

Титан (используется в нестандартных велосипедах и самолетах), хромолибден (используется в мотоциклах и автомобилях) и другие экзотические металлы имеют свои собственные проблемы термочувствительности, которые сварщики должны учитывать.

Поскольку эти металлы такие дорогие, вы не хотите ошибаться при их сварке. Следовательно, им требуется сложная машина для сварки TIG с большим количеством элементов управления, а также множество настроек и настроек, а также опытный ветеран в области управления.

Шаг 2: Установите диапазон тока, охватывающий все возможные толщины металла

Чем толще металл, тем больше силы тока, необходимой для сварки шва с хорошим проплавлением.

Поскольку стоимость сварочного аппарата частично зависит от того, сколько сока он вырабатывает, вы должны заранее определить максимальную толщину основного металла и приспособлений, с которыми вы собираетесь работать в своем цехе.

Конструкционная сталь

и трубы толщиной более полдюйма требуют использования сверхмощного сварочного аппарата MIG или аппарата для ручной сварки.Согласно Miller Electric, вам потребуется на один ампер мощности на каждую сотую дюйма мягкой стали толщиной .

Например, лист из мягкой стали толщиной 1/8 дюйма (0,125 дюйма) требует приблизительно 125 ампер. Нержавеющей стали нужно примерно на 10% меньше сока, чем углеродистой стали, а алюминию — примерно на 25% больше. Текущие настройки также привязаны к диаметру присадочной проволоки / прутков, как описано в нашем другом руководстве по настройке параметров машины MIG.

И наоборот, работа с очень тонким металлом требует небольшого нагрева (и меньшего тока) на более чувствительном сварочном аппарате.В этом случае цель состоит в том, чтобы обеспечить достаточно тепла для выполнения работы.

Иногда слабый ток вызывает нестабильную дугу, и это кошмар сварщика. Кроме того, если в основной металл поступает слишком много тепла, область вокруг сварного шва ослабевает или плавится.

Итак, если вам нужно работать с тонкой ложей или алюминием, обратите внимание и на нижнюю часть усилителя. Кроме того, многие из характеристик, только что описанных выше для алюминия, также применимы при сварке очень тонкой заготовки любого типа металла.

Если вы будете осторожны, вы можете использовать кислородно-ацетиленовый комплект для сварки тонких металлических материалов. Но убедитесь, что в горелку можно установить крошечное сварочное сопло размером три (т. Е. 000).

Шаг 3. Решите, где вы будете сваривать

Зная, где вы будете чаще всего сваривать, определите, какое оборудование вам следует приобрести.

Есть пара вещей, о которых стоит подумать:

Блок питания

Если вы подключаете машину к розетке (т.е.е., электросеть) можно выбрать следующие варианты:

  • 110/120 В переменного тока — это стандартная мощность, предоставляемая каждому потребителю коммунальной компании, жилому и коммерческому. Некоторые аппараты для дуговой сварки начального уровня рассчитаны на входную мощность 115 В, но их немного.
  • 220/240 вольт переменного тока — это высокомощная 30-амперная схема, используемая в большинстве сварочных аппаратов. Это будет доступно в любом промышленном предприятии. Другое дело жилая проводка. Поскольку для большинства сварочного оборудования требуется цепь на 30 ампер, вам, возможно, придется нанять лицензированного электрика для подключения цепи с панели управления.
  • Однофазное и трехфазное — Большая часть электрооборудования рассчитана на нормальную «однофазную» работу, питающуюся от линии напряжения 220–240, выходящей из сети. Однако на многих складах и других промышленных предприятиях доступен «трехфазный» вариант. В этом сценарии к цепи подключается третий горячий провод, обеспечивая большую силу тока для питания больших двигателей. Этот вариант также обеспечивает лучшую энергоэффективность, поэтому компании готовы вкладывать средства в трехфазные машины, чтобы сэкономить кучу денег на счетах за электроэнергию.Однако вы не можете использовать трехфазную машину дома.

Автономный сценарий

Если вы занимаетесь сваркой на открытом воздухе и у вас нет доступа к электросети, вам понадобится сварщик с приводом от двигателя или сварщик-генератор для выполнения задания. Фермеры и сварщики, работающие в поле, обычно покупают этот тип оборудования.

В зависимости от модели, генераторы работают на бензине, дизельном топливе или жидком пропане (не все три) и оснащены горелкой для ручной сварки, горелкой Tig или устройством подачи проволоки MIG / Flux-core и горелкой.Низкая цена в этой нише продуктов начинается примерно с 2000 долларов и используется только для сварки штангой.

При просмотре литературы по продукции ищите символы CC (постоянный ток) и CV (постоянное напряжение), если вы собираетесь запускать сварочный аппарат от генератора.

Аппараты

CV стоят дороже, но являются правильным выбором, если вы подключаете сварочный аппарат MIG / порошковой проволокой к генератору. Вам также необходимо знать свои требования к мощности (т. Е. Максимальную мощность), чтобы выбрать генератор правильного размера.

Beware, California разрешает использование только генераторов, отвечающих стандартам с низким уровнем выбросов углерода, также известных как CARB-совместимые.

Ветреные условия

Если вы планируете проводить сварку в незащищенных местах, где возможен ветер, это может отрицательно повлиять на ваши сварные швы.

Газ CO2 / аргон, используемый в процессе MIG для защиты сварочной ванны, выдувается ветром и неэффективен. Эта проблема приводит к окислению и пористости сварных швов.

В ветреных условиях, когда нельзя использовать защитный газ, вам может потребоваться переключить аппарат MIG в режим порошковой сварки (или использовать сварочный аппарат для прямой сварки).Проволока с покрытием обеспечивает все необходимое для хорошего шва, включая защиту, поэтому газ не требуется.

В качестве альтернативы, сварочный аппарат будет работать в прохладной окружающей среде (но не слишком ветрено). Подобно порошковой проволоке в аппарате MIG, стержневой электрод представляет собой флюсовое покрытие, обеспечивающее то, что вам нужно.

Оба эти процесса сварки содержат твердые раскислители внутри проволоки или прутка. Они испаряются прямо над лужей во время сварки, оставляя за собой защитный слой шлака.

Шаг 4. Основные характеристики для сравнения

Читая коммерческую литературу для различных сварщиков, трудно перебороть жаргонизмы и коммерческую подачу.

Но вот несколько ключевых моментов, на которые стоит обратить внимание:

Рабочий цикл

В этой спецификации указано, сколько непрерывной сварки аппарат может выполнить за десять минут.

Традиционно рабочий цикл определяется как количество минут из 10-минутного периода, в течение которого сварщик может сваривать при максимальном токе, который предлагает аппарат.После достижения предела машине необходимо дать остыть в течение оставшихся 10 минут.

Иногда производители указывают рабочий цикл в процентах. Итак, вы должны делать математику в уме. Просто умножьте процентное значение на десять, чтобы получить количество минут, которые вы можете сваривать за 10-минутный интервал. Если вы превысите рабочий цикл, машина нагреется, и цепи внутри могут поджариться.

Например, очень недорогая машина с максимальным током 70 ампер может иметь рабочий цикл 10 процентов.Это означает, что вы можете сваривать в течение 1 минуты из каждых 10 без перегрева или выгорания оборудования.

В общем, вы можете искать рабочие циклы в зависимости от ваших потребностей. Как показывает опыт:

  • легкая промышленность / любитель 20%
  • средняя 40-60%
  • сверхмощный 60-80%

Но в настоящее время производители играют с формулой. Чтобы похвастаться более высоким рабочим циклом, они сообщают процентное значение, основанное на более низкой настройке силы тока. Таким образом, для машины, которая обеспечивает рабочий цикл 10% при максимальном токе 140 ампер, вместо этого вы можете увидеть рейтинг 30% при 115 амперах.

Итак, считывая рабочий цикл для различных сварщиков, обратите внимание также на ток. Рабочий цикл увеличивается с уменьшением ампер.

С другой стороны, вы можете использовать ту же тактику, чтобы сократить рабочий цикл машины, которая в остальном соответствует всем вашим потребностям. Просто купите модель с более высоким максимальным током, чем вы ожидаете использовать. Таким образом, вы эффективно увеличиваете рабочий цикл до нужных вам усилителей.

Напряжение холостого хода

Это напряжение, исходящее от горелки или пистолета для дуговой сварки, когда ток не течет.

С одной стороны, опасно, когда на рабочем столе находится цепь под напряжением, которая может привести к серьезной травме. (Вот почему OSHA ограничивает OCV на оборудовании.)

С другой стороны, OCV влияет на работу электрода горелки при зажигании дуги. Некоторые виды сварки требуют дополнительной мощности, которую вы получаете при запуске.

В частности, для стержней E6010 и E7018 при сварке штангой требуется достаточно высокий OCV. Это позволяет получить более четкую дугу, когда сварщик царапает стержень о металл, чтобы начать сварку.

Частой проблемой для студентов является невозможность зажигания дуги, поэтому низкий OCV на небольшом сварочном аппарате может усугубить ситуацию. Вы должны принять во внимание OCV в спецификациях, если вы новичок, обучающийся сварке.

OCV около 80 вольт считается нормальным для аппарата для ручной сварки. В сварочном аппарате MIG она может упасть примерно до 35, но это не имеет большого значения, поскольку при сварке MIG дуга обычно зажигается без суеты при нажатии на курок.

Защита от тепловой перегрузки

Либо он есть в машине, либо нет, и вам следует покупать только машину, в которой он есть.Эта функция автоматически отключает выходную мощность вашего резака или пистолета, если внутренняя цепь начинает перегреваться.

Вентилятор или другой охлаждающий механизм продолжит работу, помогая рассеивать тепло (при условии, что вы оставите машину включенной).

В некоторых спецификациях эта функция четко прописана. Но с другими продуктами вам, возможно, придется обратиться к руководству по эксплуатации оборудования или обратиться к торговому представителю.

Шаг 5: Определите, нужно ли вам использовать сжатые газы

Различные газы (CO2, аргон, кислород и др.) или смеси газов используются для различных сварочных процессов.

Для сварки MIG тип газа, который вам нужен, зависит от процесса, основного металла, положения сварки и условий окружающей среды. Используемые газы включают аргон, CO2, гелий, кислород и азот или смеси этих газов. Например, обычно используется смесь аргона и CO2.

При кислородной сварке вам просто необходимы кислород и топливный газ.

Подробнее о сварочных газах можно прочитать здесь.

В аппарате TIG обычно используется чистый аргон.Но в некоторых особых случаях может потребоваться гелий. Защитный газ TIG может представлять собой специальную смесь газов, предназначенную для определенных применений, например, сварочные газы MIG.

Если вам нужно использовать сжатый газ, вы должны учитывать еще несколько вещей:

  • Если вы покупаете сварочный аппарат, для которого требуется газ, хранящийся в баллоне под давлением, в какой-то момент вам нужно будет транспортировать баллон поставщику для заправки. Убедитесь, что в вашем районе есть поставщик с разумной политикой замены и наполнения пустых резервуаров.
  • Сами резервуары можно купить или арендовать у поставщика. Цилиндры бывают разных размеров, поэтому вам нужно будет выяснить, какой размер вам нужен, исходя из того, как часто вы его будете использовать. Как правило, наполнение большого резервуара обходится не намного дороже, чем наполнение маленького. Купите самое большое, что вы можете себе позволить, и храните.
  • С газом связано множество вопросов безопасности и требований к хранению. Убедитесь, что вы понимаете, что происходит, прежде чем покупать сварочное оборудование, в котором оно используется.Если вы думаете о покупке бывшего в употреблении резервуара, убедитесь, что ваш поставщик газа согласится заполнить его перед покупкой. Всегда держите товарный чек и другую документацию под рукой. OSHA требует, чтобы все резервуары проверялись каждые два года.
  • В большинстве случаев сварки MIG с низкоуглеродистой сталью требуется сочетание 75% аргона и 25% CO2, хотя некоторые сварщики действительно используют 100% CO2 с хорошими результатами. Для сварки алюминия методами MIG и TIG обычно требуется чистый аргон. Для нержавеющей стали требуется трехкомпонентная смесь из 90% гелия, 7.5% аргона и 2,5% CO2. Не волнуйтесь; Вам не нужно смешивать газы самостоятельно. Вы просто покупаете нужную смесь, но вам понадобится отдельный баллон для каждого используемого газа.
  • Хотя использование сжатых газов в аппарате для дуговой сварки увеличивает ваши расходы, вы экономите деньги на присадочных стержнях. Присадочная проволока MIG дешевле и эффективнее электродов для сварочных аппаратов. (Самозащитная проволока с сердечником из флюса также не требует защитного газа.)
  • В настоящее время как газообразный ацетилен, так и кислород дороги.Вот почему кислородно-ацетиленовый процесс обычно используется для резки горелкой, а не для сварки.

Завершение

Выбор подходящего сварщика сводится к определению того, какая машина соответствует вашим потребностям. Пять упомянутых выше ключевых моментов помогут вам в этом процессе и включают:

  1. Определите типы металла, который вы будете сваривать.
  2. Укажите диапазон толщины, с которым вы собираетесь работать (и требуемый выходной ток).
  3. Подумайте, где вы будете работать (например,g., в помещении или на улице).
  4. Сравните ключевые характеристики сварщиков-конкурентов, например, рабочий цикл.
  5. Определитесь, нужен ли вам сжатый газ. Если да, то примите соответствующие меры.

Эти шаги не только помогут вам пройти через процесс выбора сварщика. Вы узнаете о различных единицах и о том, что лучше всего работает в определенных ситуациях. Знание различных широко используемых сварочных процессов и машин сделает вас лучшим сварщиком и повысит вашу ценность для потенциального работодателя.

Найдите своего сварщика

Мы собрали лучшие результаты в каждой категории. Найдите их ниже:

MIG Сварка нержавеющей стали

Хотя сварка нержавеющей стали может быть не такой сложной, как сварка алюминия, металл имеет свои специфические свойства, которые отличаются от обычных сталей. При сварке MIG нержавеющей стали у вас обычно есть три варианта переноса в зависимости от вашего оборудования: перенос дуги с распылением, короткое замыкание или импульсная дуга.

Распылительный перенос
Присадочные металлы для газовой дуговой сварки нержавеющей стали указаны в AWS — A5.9-93. Щелкните здесь, чтобы просмотреть полноразмерный файл Acrobat .pdf.

Электроды диаметром до 1/16 дюйма, но обычно 0,045 дюйма, 0,035 дюйма и 0,030 дюйма, используются при относительно высоких токах для создания переноса струйной дуги. Требуется ток примерно 300-350 ампер. для электрода 1/16 дюйма, в зависимости от защитного газа и типа используемой нержавеющей проволоки. Степень разбрызгивания зависит от состава и расхода защитного газа, скорости подачи проволоки и характеристик источник питания для сварки.DCEP (положительный электрод постоянного тока) используется для большинства сварочных работ с нержавеющей сталью. Для большинства дуговой сварки нержавеющих сталей рекомендуется использовать 1-2% аргоно-кислородной смеси.

На стыковых сварных швах с квадратным сечением следует использовать опорную ленту, чтобы предотвратить выпадение металла шва. При плохой подгонке или невозможности использования медной подложки просадку можно свести к минимуму сваркой короткого замыкания на первом проходе.

При сварке полуавтоматом удобно использовать приемы справа. Хотя рука оператора подвергается большему нагреву, достигается лучшая видимость.Для приварной пластины ¼ дюйма. и толще, пистолет следует двигать вперед и назад в направлении соединения и при этом немного перемещать из стороны в сторону. Однако на более тонком металле используется только возвратно-поступательное движение вдоль стыка.

Более экономичный процесс переноса с коротким замыканием для более тонкого материала следует использовать в верхнем и горизонтальном положении, по крайней мере, для корневого и первого проходов. Хотя некоторые операторы используют короткую дугу с распылением для удаления луж, сварной шов может быть чрезмерно пористым.

Короткозамкнутый переход
Блоки питания с регулировкой наклона, напряжения и индуктивности рекомендуются для сварки нержавеющей стали с короткозамкнутым переходом. В частности, индуктивность играет важную роль в обеспечении надлежащей текучести лужи.

Защитный газ, рекомендуемый для сварки коротким замыканием нержавеющей стали, содержит 90% гелия, 7,5% аргона и 2,5% двуокиси углерода. Газ обеспечивает наиболее желательный контур валика при достаточно низком уровне CO2, чтобы он не влиял на коррозионную стойкость металла.Высокая индуктивность на выходе выгодна при использовании этой газовой смеси.

Однопроходные сварные швы также можно выполнять с использованием газа аргон-CO2. CO2 в защитном газе влияет на коррозионную стойкость многопроходных сварных швов, выполненных с коротким замыканием.

Удлинитель или вылет провода должны быть как можно короче. Обратной сваркой обычно легче выполнять угловые швы, и в результате получается более аккуратный сварной шов.Для стыковых швов следует использовать сварку спереди. Наружные угловые швы можно выполнять прямолинейным движением. Следует использовать небольшое движение вперед и назад по оси сустава. Короткозамкнутые переходные швы на нержавеющей стали, выполненные с использованием защитного газа 90% He, 7-1 / 2% A, 2-1 / 2% CO2, демонстрируют хорошую коррозионную стойкость и коалесценцию. Стыковые, нахлесточные и одинарные угловые швы для материалов толщиной от 0,60 дюйма. до .125 дюйма из 321, 310, 316, 347, 304, 410 и аналогичных нержавеющих сталей могут быть успешно изготовлены.

Импульсный перенос дуги
Импульсный дуговый процесс обычно представляет собой процесс, при котором одна небольшая капля расплавленного металла переносится через дугу для каждого сильноточного импульса сварочного тока. Сильный импульс тока должен иметь достаточную величину и длительность, чтобы вызвать образование по крайней мере одной маленькой капли расплавленного металла, которая будет перемещена за счет эффекта защемления от конца проволоки к сварочной ванне. Во время слаботочной части цикла сварки дуга поддерживается, а проволока нагревается, но выделяемого тепла недостаточно для передачи металла.По этой причине продолжительность времени при низком значении тока должна быть ограничена, иначе металл будет перемещаться в глобулярном режиме.

В этом процессе чаще всего используются проволоки диаметром 0,030 дюйма, 0,035 дюйма и 0,045 дюйма. Газы для импульсной дуговой сварки — это аргон плюс 1% кислорода, такой же, как при сварке со струйной дугой. Эти и другие размеры проволоки можно сваривать. в режиме распыления при более низком среднем токе при импульсном токе, чем при непрерывном сварочном токе.Преимущество этого заключается в том, что тонкий материал можно сваривать в режиме распыления, что дает гладкий сварной шов с меньшим разбрызгиванием, чем в режиме короткого замыкания. Еще одно преимущество состоит в том, что при заданном среднем токе струйный перенос может быть получен с помощью проволоки большего диаметра. Проволока большего диаметра дешевле, чем проволока меньшего диаметра, а меньшее отношение поверхности к объему снижает возможность загрязнения сварного шва поверхностными оксидами.

Импульсная сварка MIG обеспечивает отличные характеристики при более низком токе.Этот процесс дает много преимуществ, включая низкое разбрызгивание, проплавление без протекания и удобство для оператора.

Сварка и пайка Промышленные аксессуары Вольфрамовый электрод 3,2 мм для сварки тонкой нержавеющей стали 10 шт. Сварочное оборудование ziptimberline.com

Промышленные аксессуары Вольфрамовый электрод 3,2 мм для сварки тонкой нержавеющей стали 10 шт., Промышленные аксессуары Вольфрамовый электрод 3,2 мм для сварки тонкой нержавеющей стали 10 шт. — -, ограничение по времени — скидка 50%, быстрая доставка до дверей, большой интернет-торговый центр, ежедневные предложения со скидками до 90%., Вольфрамовый электрод 3,2 мм для сварки тонкой нержавеющей стали 10 шт. Промышленные аксессуары, электрод 3,2 мм для сварки тонкой нержавеющей стали 10 шт. Промышленные аксессуары Tungsten.

Спецификация: продукт используется для сварки тонкой нержавеющей стали и подходит для большинства материалов, хорошая стабильность дуги, диаметр сердечника: приблизительно, материал: церий-вольфрам, симметричная форма волны 120-200, достаточное количество и хорошее качество. 1 дюйм, эти электроды имеют отличное зажигание дуги. AC: асимметричная форма волны 150-270.175 мм / 6, набор вольфрамовых сварочных стержней из 10 шт., Изделия из алюминия и сплавов, хорошая стабильность дуги, можно использовать в течение длительного периода, 2 мм для сварки тонкой нержавеющей стали 10 шт. — -. Промышленные аксессуары Вольфрамовый электрод 3. 3, 2, 10 стержневых вольфрамовых электродов, чувствуйте себя более безопасно, постоянный ток: подключите отрицательный полюс 220-350, используя передовую технологию бесцентрового шлифовального станка, 3, поверхность продукта имеет высокую гладкость и без заусенцев , Тип: композитная вольфрамовая игла, 2 мм / 0, 4, с использованием передовой технологии бесцентрового шлифовального станка, эти электроды имеют отличный запуск дуги.Список пакетов :, A, A, изделия из алюминия и сплавов, 5, чувствуйте себя более безопасно, Выбор сварочного тока: Дуга более концентрированная и стабильная, чем другие изделия. Изделие предназначено для сварки тонкой нержавеющей стали. Набор сварочных стержней вольфрамовых 10шт. Длина: приблизительно, A, дуга более концентрированная и стабильная, чем у других продуктов.

Добавить комментарий

Ваш адрес email не будет опубликован. Обязательные поля помечены *

Штанга рукояти 1/16 ″
1,6 мм
5/64 ″
2,0 мм
3/32 ″
2,4 мм
1/8 ″
3,2 мм
5/32 ″
4,0 мм
3/16 ″
4,8 мм
E6010
и E6011
60-90 75-130 120-165 46209 909 E6013 25-50 40-70 65-95 85-135 125-175 175-215
E7014 75-105 904 -150 135-200 185-240
E7018 70-110 90-160 130-220 170-270
E7018 — 130-160 165-225 210-290
E308L-16 25-55 50-85 70-115 95-145 135-180
ENiFe-CI — 85 85-110 110-140 120-160
E4043 50-80 70-120 110-150